• Shuffle
    Toggle On
    Toggle Off
  • Alphabetize
    Toggle On
    Toggle Off
  • Front First
    Toggle On
    Toggle Off
  • Both Sides
    Toggle On
    Toggle Off
  • Read
    Toggle On
    Toggle Off
Reading...
Front

Card Range To Study

through

image

Play button

image

Play button

image

Progress

1/115

Click to flip

Use LEFT and RIGHT arrow keys to navigate between flashcards;

Use UP and DOWN arrow keys to flip the card;

H to show hint;

A reads text to speech;

115 Cards in this Set

  • Front
  • Back

is it ok for the cops to read Miranda warnings to a guy they arrested, then get a confession of burglary without letting the guy know the person he hit over the head is in critical condition (and the guy might end up charged with murder)?

yes - no duty to reveal that info

A plaintiff sued a defendant for a libelous letter received by an investigator. The authenticity and contents of the letter are disputed. The investigator, if permitted, will testify that, "I received a letter that I cannot now find, which read: 'Dear [investigator], You inquired about [the plaintiff]. We fired him last month when we discovered that he had been stealing from the stockroom. [the defendant]'."

The testimony should be admitted in evidence only if


A. the jury finds that the investigator has quoted the letter precisely.B. the jury is satisfied that the original letter is unavailable.C. the judge is satisfied that the investigator has quoted the letter precisely.D. the judge finds that the original letter is unavailable.

Answer choice D is correct. The contents of the letter are being offered as nonhearsay, as a statement by an opposing party(the defendant). Because the contents of the letter are disputed, however, the testimony as to its contents must satisfy the best evidence rule, which requires the original letter to be unavailable. Answer choices A and B are incorrect because admissibility of evidence is a decision made by the judge, not the jury. Answer choice C is incorrect because the credibility of the investigator's memory is determined by the jury.

who holds the dr/patient privilege?

the patient

what kind of change of circumstances needs to happen for an implied reciprocal servitude to no longer be effective?

drastic - difficulty selling the plots is not enough

On March 1, a landowner and a builder orally agreed that the builder would erect a boathouse on the landowner's lot and dig a channel from the boathouse, across a neighbor's lot, to a lake. The neighbor had already orally agreed with the landowner to permit the digging of the channel across the neighbor's lot. The builder agreed to begin work on the boathouse on March 15, and to complete all the work before June 1. The total price of $10,000 was to be paid by the landowner in three installments: $2,500 on March 15; $2,500 when the boathouse was completed; $5,000 when the builder finished the digging of the channel. Assume that the landowner tendered the $2,500 on March 15, and that the builder refused to accept it or to perform.

In an action by the landowner against the builder for breach of contract, which of the following can the builder successfully use as a defense?


A. The landowner-builder agreement permitting the digging of the channel across the neighbor's lot was not in writing.B. The landowner-builder agreement was not in writing.C. The landowner needs an easement to conduct work on the neighbor's property.D. The neighbor refused to permit access to the builder.

Answer choice D is correct. The doctrine of frustration of purpose applies when unexpected events arise to make a contract impossible to be performed, entitling the frustrated party to rescind the contract without paying damages. If the neighbor refuses to grant access, the builder's work is unexpectedly frustrated, creating an impossibility of his performance. Answer choice A is incorrect because it assumes that the digging of the channel requires a writing, which it does not here, because the neighbor orally agreed to the construction. Answer choice B is incorrect because it incorrectly assumes that an interest in property has been conveyed, requiring the Statute of Frauds to be invoked. Answer choice C is incorrect because the neighbor has already agreed to the construction.

An owner's car sustained moderate damage in a collision with a car operated by a driver. The accident was caused solely by the driver's negligence. The owner's car was still drivable after the accident. Examining the car the next morning, the owner could see that a rear fender had to be replaced. He also noticed that gasoline had dripped onto the garage floor. The collision had caused a small leak in the gasoline tank.
The owner then took the car to a mechanic, who owns and operates a body shop, and arranged with the mechanic to repair the damage. During their discussion the owner neglected to mention the gasoline leakage. Thereafter, while the mechanic was loosening some of the damaged material with a hammer, he caused a spark, igniting vapor and gasoline that had leaked from the fuel tank. The mechanic was severely burned.
The mechanic has brought an action to recover damages against the owner and the driver. The jurisdiction has adopted a pure comparative negligence rule in place of the traditional common-law rule of contributory negligence.
In this action, will the mechanic obtain a judgment against the driver?


A. No, unless there is evidence that the driver was aware of the gasoline leak.B. No, if the mechanic would not have been harmed had the owner warned him about the gasoline leak.C. Yes, unless the mechanic was negligent in not discovering the gasoline leak himself.D. Yes, if the mechanic's injury was a proximate consequence of the driver's negligent driving.

Answer choice D is correct. The facts state that the driver was negligent. If that negligence was the actual and proximate cause of an injury, then the driver is liable. Clearly, the mechanic's injury would not have happened but for the driver's negligent operation of the car. Therefore, so long as the driver's operation of the car was also the proximate cause of the mechanic's injury, then the driver will be liable. Answer choice A is incorrect because a plaintiff is liable for all damages for which his negligence was the actual and proximate cause; there is no rule that the plaintiff must be aware of those damages. Answer choice B is incorrect because the owner's failure to warn the mechanic was foreseeable negligence, which is not a superseding cause, and therefore does not relieve the driver of liability. Answer choice C is incorrect because, in a pure comparative negligence jurisdiction, the mechanic's recovery would be proportionately reduced based on his own negligence, but would not be barred.

An owner had title to Greenview in fee simple. Without the owner's knowledge, a neighbor entered Greenview in 1980 and began to graze a flock of sheep he owned. After twelve years of possession of Greenview, the neighbor gave possession of Greenview to a buyer. At the same time, the neighbor also purported to transfer his sheep and all his interests in the land to the buyer by a document that was sufficient as a bill of sale to transfer personal property but was insufficient as a deed to transfer real property. One year later, the buyer entered into a lease with the owner to lease Greenview for a period of five years. After the end of the five-year term of the lease, the buyer remained on Greenview for an additional three years and then left Greenview. At that time, the owner conveyed Greenview by a quitclaim deed to a purchaser. The period of time to acquire title by adverse possession in the jurisdiction is ten years.

After the owner's conveyance to the purchaser, title to Greenview was in


A. the neighbor.B. the owner.C. the buyer.D. the purchaser.

Answer choice A is correct. For adverse possession to ripen into title, possession must be continuous, open and notorious, actual, exclusive, and hostile. The neighbor occupied Greenview for 12 years, grazed sheep on it, he did not share ownership with the owner, and his intent was to claim ownership as evidenced by the fact that he later attempted to give possession of Greenview to the buyer. Answer choice B is incorrect, because the use of the neighbor clearly indicates that adverse possession has taken place. Answer choice C is incorrect because the document the neighbor gave to the buyer was insufficient to transfer real property and the buyer did not satisfy the requirements of adverse possession on his own. Answer choice D is incorrect because the owner no longer has title to pass to the purchaser. The foregoing NCBE MBE question has been modified to reflect current NCBE stylistic approaches; the NCBE has not reviewed or endorsed this modification.


In the plaintiff's personal injury action, a physician who had no previous knowledge of the matter sat in court and heard all the evidence about the plaintiff's symptoms and conditions. The physician is called to give her opinion whether the plaintiff's injuries are permanent.

May the physician so testify?


A. Yes, provided she first identifies the data on which her opinion is based.B. Yes, because an expert may base her opinion on facts made known to her at the trial.C. No, because she has no personal knowledge of the plaintiff's condition.D. No, because permanence of injury is an issue to be decided by the jury.

Answer choice B is correct. A witness qualified as an expert by the court can offer an opinion based on her personal observations, facts made known during trial, or other reliable facts known outside of the courtroom. Here, the physician can testify based on her qualification as a medical expert and based on the facts she heard in court. Thus, answer choice C is incorrect. Answer choice A is incorrect because here, the expert's opinion is based on facts made known to her during trial. Answer choice D is incorrect because it misstates the rule regarding an expert's opinion on the ultimate issue in a case. An expert may testify to ultimate issues in a case, and the jury decides how much weight to accord the testimony.

A defendant was tried for reckless driving following a hit-and-run accident the defendant denies having caused. The defendant called a witness to testify to alibi. On cross-examination of the witness, the prosecutor asked, "Weren't you on the jury that acquitted the defendant of another criminal charge?"

The best reason for sustaining an objection to this question is that


A. the question goes beyond the scope of direct examination.B. the probative value of the answer would be outweighed by its tendency to mislead.C. the question is leading.D. prior jury service in a case involving a party renders the witness incompetent.

Answer choice B is correct. Although the question is relevant and may expose a potential bias on the part of the witness, the jury may be misled when it hears about a prior criminal charge. Answer choice A is incorrect because on cross-examination the credibility of a witness may be examined, even if it was not directly discussed during direct examination. Answer choice C is incorrect because leading questions are permitted on cross-examination. Answer choice D is incorrect because, although a juror may not testify at a trial in which he is sitting, prior service does not make the witness incompetent. The foregoing NCBE MBE question has been modified to reflect current NCBE stylistic approaches; the NCBE has not reviewed or endorsed this modification.

In 1990, the owner of a 300-acre tract, prepared and duly recorded a subdivision plan called Quiet Meadows. The plan showed 90 three-acre lots and a thirty-acre tract in the center that was designated "Future Public School." The owner published and distributed a brochure promoting Quiet Meadows which emphasized the proximity of the lots to the school properly and indicated potential tax savings "because the school district will not have to expend tax money to acquire this property." There is no specific statute concerning the dedication of school sites. The owner sold 50 of the lots to individual purchasers. Each deed referred to the recorded plan. In 1996, the owner sold the remaining 40 lots and the thirty-acre tract to a buyer by a deed that referred to the plan. The buyer sold the 40 lots to individual purchasers and the thirty-acre tract to a restaurateur. None of the deeds from the buyer referred to the plan. The school board of the district in which Quiet Meadows is situated has voted to erect a new school on the thirty-acre tract.

In an appropriate action between the school board and the restaurateur to determine title, the result will be in favor of


A. the restaurateur, because the school board has been guilty of laches.B. the restaurateur, because his deed did not refer to the subdivision plan.C. the school board, because the restaurateur had constructive notice of the proposed use of the tract.D. the school board, because there has been a dedication and acceptance of the tract.

Answer choice D is correct. Because there is no statute governing dedication for schools, the owner's deed is sufficient to establish a dedication of land for public use. Answer choice A is incorrect because there was no unreasonable delay in asserting their rights. Answer choice B is incorrect because the prior deed referred to the recorded plan containing the dedication. Answer choice C is incorrect because, even though the restaurateur had notice of the proposed use, this answer choice does not establish why the school board is the rightful owner. The foregoing NCBE MBE question has been modified to reflect current NCBE stylistic approaches; the NCBE has not reviewed or endorsed this modification.


A borrower owed a lender $5,500, plus interest at 9.25% until paid, on an overdue promissory note, collection of which would become barred by the statute of limitations on November 30. On the preceding July 15, the borrower and the lender both signed a writing in which the borrower promised to pay the note in full on the following February 28, plus interest at 9.25% until that date, and the lender promised not to sue on the note in the meantime. The lender, having received some advice from her non-lawyer friend, became concerned about the legal effect of the July 15 agreement. On December 26, acting pro se as permitted by the rules of the local small claims court, she filed suit to collect the note.

Assuming that there is no controlling statute, is the July 15 agreement an effective defense for the borrower?


A. Yes, because the borrower's promise to pay interest until February 28 was consideration for the lender's promise not to sue.B. Yes, because the law creates a presumption that the borrower relied on the lender's promise not to sue.C. No, because there was no consideration for the lender's promise not to sue, in that the borrower was already obligated to pay $5,500 plus interest at 9.25% until the payment date.D. No, because the borrower's July 15 promise is enforceable with or without consideration.

Answer choice A is correct. As the statute of limitations would apply as of November 30 to bar any requirement of payment by borrower, the borrower's new promise to pay interest until February 28 was proper consideration for the lender's promise not to sue on the original agreement. Answer choice B is incorrect, as there is no such presumption under the law. Answer choice C is incorrect, as the borrower's obligation would have been cut off as of November 30 because of the statute of limitations. Thus, the new promise by the borrower constituted proper consideration for the lender's promise not to sue. Answer choice D is incorrect, as a contractual modification under the common law generally must be supported by consideration. The foregoing NCBE MBE question has been modified to reflect current NCBE stylistic approaches; the NCBE has not reviewed or endorsed this modification.

The plaintiff and her brother owned in fee simple Greenacre as tenants in common, each owning an undivided one-half interest. The plaintiff and her brother joined in mortgaging Greenacre to the defendant by a properly recorded mortgage that contained a general warranty clause. The brother became disenchanted with land-owning and notified the plaintiff that he would no longer contribute to the payment of installments due the defendant. After the mortgage was in default and the defendant made demand for payment of the entire amount of principal and interest due, the plaintiff tendered to the defendant, and the defendant deposited, a check for one-half of the amount due the defendant. The plaintiff then demanded a release of the plaintiff's undivided one-half interest. The defendant refused to release any interest in Greenacre. The plaintiff promptly brought an action against the defendant to quiet title to an undivided one-half interest in Greenacre.

In such action, the plaintiff should


A. lose, because the defendant's title had been warranted by an express provision of the mortgage.B. lose, because there was no redemption from the mortgage.C. win, because the plaintiff is entitled to marshalling.D. win, because the co-tenancy of the mortgagors was in common and not joint.

Answer choice B is correct. As tenants in common, both the plaintiff and her brother are jointly and severally liable for the entire mortgage, and the defendant can collect from either or both. Answer choice A is incorrect because the fact that title was warranted by a provision in the mortgage is irrelevant to whether the plaintiff is liable on the mortgage. Answer choice C is incorrect because marshalling does not apply to this situation. Answer choice D is incorrect because it confuses the concept of "joint tenancy" with holding the mortgage "jointly and severally."


A landscaper was charged with the murder of a man who had been strangled and whose body was found in some woods near his home. The landscaper suffers from a neurological problem that makes it impossible for him to remember an occurrence for longer than 48 hours.

After the landscaper was charged, the police visited him and asked if they might search his home. The landscaper consented. The police found a diary written by the landscaper.

An entry dated the same day as the victim's disappearance read, "Indescribable excitement. Why did no one ever tell me that killing gave such pleasure to the master?"
The landscaper was charged with murder. His attorney has moved to exclude the diary from evidence on the ground that its admission would violate the landscaper's privilege against self-incrimination. Counsel has also argued that the landscaper could not give informed consent to the search because more than 48 hours had passed since the making of the entry and hence he could not remember the existence of the incriminating entry at the time he gave his consent. There is no evidence that the police officers who secured the landscaper's consent to the search were aware of his memory impairment.

With regard to the diary, the court should:


A. admit it, because the landscaper's consent was not obtained by intentional police misconduct and the landscaper was not compelled to make the diary entry.B. admit it, pursuant to the good-faith exception to the exclusionary rule.C. exclude it, because the landscaper was not competent to consent to a search.D. exclude it, because use of the diary as evidence would violate the landscaper's privilege against self-incrimination.

Answer choice A is correct. Consent to a search is voluntary if it is made free from threat or compulsion. The facts indicate that was the case here, so the search of the landscaper's home was valid. The Fifth Amendment prohibits the government from compelling self-incriminating testimony. The diary entry was made freely, without government compulsion, and the landscaper consented to the search that led the police to find the diary; the landscaper's inability to remember writing it does not negate its voluntary nature, making answer choices C and D incorrect. Answer choice B is incorrect because the good-faith exception applies to illegal searches, which is not the case here, as the landscaper consented.

atty-client privilege applies to advice a client seeks to commit a crime only when the atty knows what the purpose is or even if the atty didn't know what the client's purpose was in seeking the advice?

be careful! doesn't matter what the atty knew, only what the client intended

can you plead the 5th regarding illegal activity in both a civil and a criminal case (if you're the witness and the case isn't about you)?

yes

a police plan to randomly stop cars and check license and registration in a high crime area of the city in order reduce crime is....

illegal - have to have PC that a crime has been committed or is being committed (or at least a reasonable suspicion for a Terry stop)

A drug dealer was prosecuted for selling cocaine to an undercover police agent. At his trial, he testified that he only sold the drugs to the agent, whom the drug dealer knew as "Speedy," because Speedy had told him that he (Speedy) would be killed by fellow gang members unless he supplied them with cocaine. The prosecution did not cross-examine the drug dealer. As rebuttal evidence, however, the prosecutor introduced records, over the drug dealer's objection, showing that the drug dealer had two prior convictions for narcotics-related offenses. The court instructed the jury concerning the defense of entrapment and added, also over the drug dealer's objection but in accord with state law, that it should acquit on the ground of entrapment only if it found that the defendant had established the elements of the defense by a preponderance of the evidence. The drug dealer was convicted.

On appeal, the drug dealer's conviction should be:


A. reversed, because it was an error for the court to admit the evidence of his prior convictions as substantive evidence.B. reversed, because it was a violation of due process to impose on the defense a burden of persuasion concerning entrapment.C. reversed, for both of the above reasons.D. affirmed, because neither of the above reasons constitutes a ground for reversal.

Answer choice D is correct. Neither choice A nor choice B constitutes a ground for reversal. Answer choice A is incorrect because a defendant's lack of predisposition toward committing a crime is an element of entrapment. Therefore, evidence of the defendant's prior convictions was properly admitted to rebut his defense. Answer choice B is incorrect because it is not violative of Due Process for the state to place the burden of proving an affirmative defense on the defense. Because answer choices A and B are incorrect, answer choice C also is incorrect.

A rancher and a farmer own adjacent tracts of rural land. For the past nine years, the rancher has impounded on her land the water that resulted from rain and melting snow, much of which flowed from the farmer's land. The rancher uses the water in her livestock operation. Recently, the farmer increased the size of his farming operation and built a dam on his land near the boundary between the two tracts. Because of the dam, these waters no longer drain from the farmer's land onto the rancher's land. There is no applicable statute. The rancher sued the farmer to restrain him from interfering with the natural flow of the water onto her land.

Who is likely to prevail?


A. The farmer, because he has the right to use all of the water impounded on his land.B. The farmer, because the rancher's past impoundment of water estops her from asserting the illegality of the farmer's dam.C. The rancher, because she has acquired riparian rights to use the water.D. The rancher, because the farmer is estopped to claim all of the surface water on his land.

Answer choice A is correct. This water is surface water (also called diffused surface water). Although there are different views regarding the way an owner may expel such water, an owner such as the farmer, may impound such water, especially in the absence of any malice. Answer choice B is incorrect because, in the absence of malice, either landowner may impound diffuse surface waters. The doctrine of estoppel does not apply as neither impoundment is illegal. Answer choice C is incorrect because riparian waters are waters with defined beds and banks, such as streams, rivers, and lakes. Waters from melting snows and rain is surface water. A riparian owner is one whose land borders such waters. Here, the only water at issue is surface water. Answer choice D is incorrect because either landowner may impound surface water. The farmer's impoundment did not run counter to any past representation relied upon by the rancher. In these circumstances, the doctrine of estoppel does not apply, and the farmer will prevail.

One evening, the defendant had several drinks and then started to drive home. As he was proceeding down a boulevard, an automobile pulled out of a side street to his right. The defendant's car struck this automobile broadside. The driver of the other car was killed as a result of the collision. A breath-analysis test administered after the accident showed that the defendant satisfied the legal definition of intoxication.

If the defendant is prosecuted for manslaughter, his best chance for acquittal would be based on an argument that:


A. the other driver was contributorily negligent.B. the collision would have occurred even if the defendant had not been intoxicated.C. because of his intoxication he lacked the mens rea needed for manslaughter.D. driving while intoxicated requires no mens rea and so cannot be the basis for misdemeanor manslaughter.

Answer choice B is correct. Intoxication is not a valid defense to a general intent crime such as manslaughter, so the defendant's best argument would be that something other than his intoxication caused the collision. Answer choice A is incorrect because the defendant could still have been criminally responsible for the collision due to his own recklessness, regardless of the other driver's liability. Answer choice C is incorrect because the defendant's recklessness in driving while intoxicated actually is the requisite mens rea for manslaughter. Answer choice D is incorrect because the defendant could be convicted of criminally negligent manslaughter regardless of whether misdemeanor manslaughter is proven.


An independent municipal water-supply district incorporated under the applicable laws of the state in which it sits. The district was created solely to supply water to an entirely new community in a recently developed area of the state. That new community has never engaged in any discrimination against members of minority groups.
The five-member elected governing board of the water district contains two persons who are members of racial minority groups. At its first meeting, the governing board adopted a rule unqualifiedly setting aside 25% of all positions on the staff of the district and 25% of all contracts to be awarded by the district to members of racial minority groups. The purpose of the rule was "to help redress the historical discrimination against these groups in this country and to help them achieve economic parity with other groups in our society." A suit by appropriate parties challenges the constitutionality of these set-asides.
Assuming that no federal statute applies, is the challenge to the set-aside likely to succeed?


A. Yes, because the set-asides would deny other potential employees or potential contractors the equal protection of the laws.B. Yes, because the set-asides would impermissibly impair the right to contract of other potential employees or potential contractors.C. No, because the set-asides would assure members of racial minority groups the equal protection of the laws.D. No, because the function and activities of the water district are of a proprietary nature rather than a governmental nature and, therefore, are not subject to the usual requirements of the Fourteenth Amendment.

Answer choice A is correct. In order for an affirmative action program to survive, the government must show more than a history of societal discrimination. The government must be guilty of specific past discrimination against the group it is seeking to favor, and the remedy must be narrowly tailored to end that discrimination and eliminate its effects. The water district’s attempt to remedy general societal injustice, rather than specific acts of discrimination, is not a compelling state interest, and does not survive strict scrutiny. Accordingly, the challenge is likely to succeed. Answer choice B is incorrect because a claim of impairment of contracts applies only to existing contracts, and not to those not yet entered into. Answer choice C is incorrect because the water district did not engage in past discrimination, and its attempt to remedy general societal discrimination through set-asides is impermissible. Answer choice D is incorrect because the water district is a municipal (i.e., local government) agency, and the 14th Amendment would apply. The foregoing NCBE MBE question has been modified to reflect current NCBE stylistic approaches; the NCBE has not reviewed or endorsed this modification.

A plaintiff has sued a defendant for personal injuries the plaintiff suffered when she was bitten as she was trying to feed a rat that was part of the defendant's caged-rat experiment at a science fair. At trial, the plaintiff offers evidence that immediately after the incident the defendant said to her, "I'd like to give you this $100 bill, because I feel so bad about this."
Is the defendant's statement admissible?


A. No, because it is not relevant to the issue of liability.B. No, because it was an offer of compromise.C. Yes, as a present sense impression.D. Yes, as the statement of a party-opponent

Answer choice D is correct. An out-of-court statement by a party that is relevant to his or her liability is admissible under the exception to the hearsay rule for statements of a party-opponent. One might think that the statement would be excluded because of Rule 408, which excludes statements that are made to settle a claim. But that rule is inapplicable, because it applies only when the statement is made to compromise a disputed claim. Here, at the time the defendant made the statement, he was not contesting that he was at fault. Therefore, there was no disputed claim. Accordingly, answer choice B is incorrect. Answer choice A is incorrect because the defendant's statement of contrition and offer of compensation clearly have a tendency to prove that he is liable, and a tendency is all that is required for the evidence to be relevant under Rule 401. The statement is admissible as the statement of a party-opponent. Answer choice C is incorrect because the exception to the hearsay rule for present sense impressions covers a statement describing or explaining an event or condition made during or immediately after the event or condition. The defendant's statement is just an expression of contrition and not an attempt to explain any event or condition. However, the statement is admissible as the statement of a party-opponent.


A company, Fixtures, Inc., in a signed writing, contracted with a buyer, Apartments, for the sale of 50 identical sets of specified bathroom fixtures, 25 sets to be delivered on March 1, and the remaining 25 sets on April 1. The agreement did not specify the place of delivery, or the time or place of payment. On March 1, the company tendered 24 sets to the buyer and explained, "One of the 25 sets was damaged in transit from the manufacturer to us, but we will deliver a replacement within 5 days."

Which of the following statements is correct?


A. The buyer is entitled to accept any number of the 24 sets, reject the rest, and cancel the contract both as to any rejected sets and the lot due on April 1.B. The buyer is entitled to accept any number of the 24 sets and to reject the rest, but is not entitled to cancel the contract as to any rejected sets or the lot due on April 1.C. The buyer must accept the 24 sets but is entitled to cancel the rest of the contract.D. The buyer must accept the 24 sets and is not entitled to cancel the rest of the contract.

Answer choice D is correct. In an installment contract, if the seller makes a non-conforming tender or tenders non-conforming goods under one segment of the contract, the buyer can reject only if the non-conformity substantially impairs the value of that shipment to the buyer and cannot be cured. In this case, only one of the 25 sets was damaged, making the non-conformity insubstantial. Answer choice A is incorrect because although a buyer would usually be able to reject all, part, or none of a non-conforming tender, the U.C.C. does not allow the buyer to do so in installment contracts. Answer choice B is incorrect, as the buyer must accept the shipment if the seller gives adequate assurances that the non-conformity will be cured within a reasonable timeframe. The company's promise to deliver within 5 days serves as adequate assurances that it will cure the non-conformity. Answer choice C is incorrect because the non-conformity of goods did not substantially impair the buyer, and the buyer received adequate assurances of cure.

IF you condition your purchase of a home (expressly in the contract) on getting financing, do you have to apply for mortgages?

yes - good faith requirement

The United States had long recognized the ruling faction in a foreign country as that country's government, despite an ongoing civil war. Throughout the civil war, the ruling faction controlled the majority of the country's territory, and the United States afforded diplomatic immunity to the ambassador representing the ruling faction.
A newly elected President of the United States decided to recognize a rebel group as the government of the foreign country and notified the ambassador from the ruling faction that she must leave the United States within 10 days. The ambassador filed an action in federal district court for a declaration that the ruling faction was the true government of the foreign country and for an injunction against enforcement of the President's order that she leave the United States. The United States has moved to dismiss the action.
If the court dismisses the action, what will be the most likely reason?


A. The action involves a nonjusticiable political question.B. The action is not ripe.C. The action is within the original jurisdiction of the U.S. Supreme Court.D. The ambassador does not have standing.

Answer choice A is correct. The action likely satisfies the political question doctrine and therefore should be dismissed as nonjusticiable. The President's Article II power to receive foreign ambassadors is likely a textually demonstrable commitment by the Constitution of exclusive authority to recognize foreign governments. Moreover, Article II provides no judicially manageable standards by which a court could review the constitutionality of a President's decision on whether to recognize a foreign government. Finally, because the action involves the President's administration of foreign affairs, the prudential elements of the political question doctrine also indicate that the court should dismiss the action as nonjusticiable. Answer choice B is incorrect. The action is ripe for adjudication even though the ambassador may remain in the United States for 10 days. The ambassador has suffered immediate harm because she no longer represents the foreign country in the United States, she has lost her diplomatic immunity, and she is facing expulsion within a very short period of time. Also, the constitutional issues are fit for review without waiting for the ambassador's expulsion. Answer choice C is incorrect. Although Article III of the Constitution provides that the Supreme Court has original jurisdiction over actions involving ambassadors, federal district courts also may exercise original jurisdiction over actions within the Supreme Court's original jurisdiction. Answer choice D is incorrect. The ambassador has standing, because she has been injured by the President's decision that her faction is no longer the government of her country, her injury is fairly traceable to this decision, and the injury is likely redressable by a court order invalidating the decision.


The owner of Blackacre, a residential lot improved with a dwelling, conveyed it for a valuable consideration to a buyer. The dwelling had been constructed by a prior owner. The buyer had inspected Blackacre prior to the purchase and discovered no defects. After moving in, the buyer became aware that sewage seeped into the basement when the toilets were flushed. The original owner said that this defect had been present for years and that he had taken no steps to hide the facts from the buyer. The buyer paid for the necessary repairs and brought an appropriate action against the original owner to recover his cost of repair.

If the buyer wins, it will be because


A. the owner failed to disclose a latent defect.B. the buyer made a proper inspection.C. the situation constitutes a health hazard.D. the owner breached the implied warranty of habitability and fitness for purpose.

Answer choice A is correct. Because the buyer inspected the premises for patent defects, the only way he could recover for the damage is if the defect was latent and the owner failed to disclose it. Answer choice B is incorrect because the buyer's inspection would only protect the owner from having to disclose patent defects, and the call of the question assumes the buyer won. Answer choice C is incorrect because a patent health hazard would not make the owner responsible, only a latent defect would. Answer choice D is incorrect because a warranty of habitability is implied when property is rented, not when it is sold. Fitness for a particular purpose applies to the sale of goods, and is inapplicable to the sale of property.


An automobile retailer had an adult daughter who needed a car in her employment but had only $6,000 with which to buy one. The retailer wrote to her, "Give me your $6,000, and I'll give you the car on our lot that we have been using as a demonstrator." The daughter thanked her father and paid him the $6,000. As both the retailer and his daughter knew, the demonstrator was reasonably worth $20,000. After the daughter had paid the $6,000, but before the car had been delivered to her, one of the retailer's sales staff sold and delivered the same car to a customer for $20,000. Neither the salesperson nor the customer was aware of the transaction between the retailer and the daughter.

Does the daughter, after rejecting a tendered return of the $6,000 by the retailer, have an action against him for breach of contract?


A. Yes, because the retailer's promise was supported by bargained-for consideration.B. Yes, because the retailer's promise was supported by the moral obligation a father owes his child as to the necessities of modern life.C. No, because the payment of $6,000 was inadequate consideration to support the retailer's promise.D. No, because the salesperson's delivery of the car to the customer made it impossible for the retailer to perform.

Answer choice A is correct. By paying the retailer the $6,000, something she was not otherwise obligated to do, the daughter gave bargained-for consideration for the retailer's promise to deliver the car. The fact that they both knew that $6,000 was far less than the reasonable value of the car is irrelevant, as courts will not inquire into the value of consideration. Answer choice B is incorrect, as even though a father has an obligation to provide for his child, a car is not a necessity. Answer choice C is incorrect, as the $6,000 was a bargained-for legal detriment. A court will not inquire into the value of consideration. Answer choice D is incorrect, as it was the retailer's employee's fault that the car was sold to someone else, and the retailer would be held responsible. This is not a situation where the car was destroyed through no fault of the retailer or a law was passed that makes it illegal to sell the car.

The owner in fee simple of a small farm consisting of thirty acres of land improved with a house and several outbuildings leased the same to a tenant for a ten-year period. After two years had expired, the government condemned twenty acres of the property and allocated the compensation award to the owner and the tenant according to their respective interest so taken. It so happened, however, that the twenty acres taken embraced all of the farm's tillable land, leaving only the house, outbuildings, and a small woodlot. There is no applicable statute in the jurisdiction where the property is located, nor any provision in the lease relating to condemnation. The tenant quit possession, and the owner brought suit against him to recover rent.

The owner will


A. lose, because there has been a frustration of purpose which excuses the tenant from further performance of his contract to pay rent.B. lose, because there has been a breach of the implied covenant of quiet enjoyment by the owner's inability to provide the tenant with possession of the whole of the property for the entire term.C. win, because of the implied warranty on the part of the tenant to return the demised premises in the same condition at the end of the term as they were at the beginning.D. win, because the relationship of landlord and tenant was unaffected by the condemnation, thus leaving the tenant still obligated to pay rent.

Answer choice D is correct. Because a tenant must pay a reasonable rental value of the premises occupied if there has been a partial eviction by a third party with a superior claim to the property, the tenant must pay rent for the remaining property. Answer choice A is incorrect. Frustration of purpose occurs when the entire purpose of the contract has been destroyed. Because it is unclear if the tenant was farming on the land or contracted the land for some other purpose, answer choice A cannot be chosen. Answer choice B is incorrect because the owner is not responsible for the partial eviction. Answer choice C is incorrect because the majority view today is that the tenant is excused from paying rent if the premises are destroyed, as long as the tenant is not at fault for the destruction.

A grantor conveyed her only parcel of land to a grantee by a duly executed and delivered warranty deed, which provided: "To have and to hold the described tract of land in fee simple, subject to the understanding that within one year from the date of the instrument said grantee shall construct and thereafter maintain and operate on said premises a public health center." The grantee constructed a public health center on the tract within the time specified and operated it for five years. At the end of this period, the grantee converted the structure into a senior citizens' recreational facility. It is conceded by all parties in interest that a senior citizens' recreational facility is not a public health center. In an appropriate action, the grantor seeks a declaration that the change in the use of the facility has caused the land and structure to revert to her.

In this action, the grantor should


A. win, because the language of the deed created a determinable fee, which leaves a possibility of reverter in the grantor.B. win, because the language of the deed created a fee subject to condition subsequent, which leaves a right of entry or power of termination in the grantor.C. lose, because the language of the deed created only a contractual obligation and did not provide for retention of property interest by the grantor.D. lose, because an equitable charge is enforceable only in equity.

Answer choice C is correct. Answer choice A is incorrect because the deed did not create a determinable fee. There was no durational language such as "so long as," "while," "during," or "until." Answer choice B is incorrect because the deed did not create a fee simple subject to a condition subsequent. There was no conditional language such as "provided that," "on condition that," or "but if." Thus, the deed did not retain any interest in the grantor. Answer choice D is incorrect because this is not an equitable charge.

A customer pledged a stock certificate to a bank as security for a loan. A year later, when the customer fully repaid the loan, the bank refused the customer's demand to return the stock certificate because the officer dealing with the loan had the mistaken belief that there was still a balance due. No one at the bank reviewed the records until two months later, at which time the error was discovered. The bank then offered to return the stock certificate. However, the customer refused to accept it.
At the time the customer pledged the certificate, the shares were worth $10,000; at the time the customer repaid the loan, the shares were worth $20,000; and at the time the bank offered to return the certificate, the shares were worth $5,000.
If the customer brings an action against the bank based on conversion, how much, if anything, should the customer recover?


A. Nothing, because the bank lawfully came into possession of the certificate.B. $5,000, because that was the value of the shares when the customer refused to accept the certificate back.C. $10,000, because that was the value of the shares when the bank came into possession of the certificate.D. $20,000, because that was the value of the shares when the customer was entitled to the return of the certificate.

Answer choice D is correct. Conversion is equivalent to a forced sale of the chattel to the defendant, who is liable for the full value of the chattel at the time of the tort. The tort occurred when the bank refused to relinquish the stock certificate in response to the customer's lawful demand, and at that time the shares were worth $20,000. Answer choice A is incorrect. Someone who refuses to surrender a chattel to another person who is entitled to its immediate possession is liable for conversion even if the one holding the chattel originally came into possession lawfully. The bank is liable to the customer for the value of the shares at the time the bank refused the customer's lawful demand for return of the certificate - $20,000. Answer choices B and C are incorrect because the defendant is liable for the full value of the chattel at the time of the tort - when the bank refused to relinquish the stock certificate in response to the customer's lawful demand, and at that time the shares were worth $20,000.

Police received information from an undercover police officer that she had just seen two men (whom she described) in a red pickup truck selling marijuana to schoolchildren near the city's largest high school. A few minutes later, two police officers saw a pickup truck fitting the description a half block from the high school. The driver of the truck matched the description of one of the men described by the undercover officer. The only passenger was a young woman who was in the back of the truck. The police saw her get out and stand at a nearby bus stop. They stopped the truck and searched the driver. In the pocket of the driver's jacket, the police found a small bottle of pills that they recognized as narcotics. They then broke open a locked toolbox attached to the flatbed of the truck and found a small sealed envelope inside. They opened it and found marijuana. They also found a quantity of cocaine in the glove compartment. After completing their search of the driver and the truck, the police went over to the young woman and searched her purse. In her purse, they found a small quantity of heroin. Both the driver and the young woman were arrested and charged with unlawful possession of narcotics.

If the young woman moves to suppress the use as evidence of the heroin, the court should:


A. grant the motion, because she did not fit the description given by the informant and her mere presence does not justify the search.B. grant the motion, because the police should have seized her purse and then obtained a warrant to search it.C. deny the motion, because she had been a passenger in the truck and the police had probable cause to search the truck.


D. deny the motion, because she was planning to leave the scene by bus and so exigent circumstances existed.

Answer choice A is correct. A search or seizure is constitutional if it is pursuant to a warrant based on probable cause or if it falls under an exception to the warrant requirement. In this case, the police had no warrant to search the young woman. Nor did they have probable cause to arrest her and search her incident to that arrest -- there is no evidence that she sold marijuana or was in the truck when it was sold. Answer choice B is incorrect because the police had no warrant or probable cause to seize the purse, so the seizure would be illegal even if they waited for a warrant to search the purse. Answer choice C is incorrect because, while the police did have probable cause to search the truck, she was not in the truck at the time she was searched. Her motion is to suppress evidence found in her purse, which was not in the truck at the time the police searched it. Answer choice D is incorrect because there could be no exigency if the police lacked probable cause to believe the woman was involved in any illegal conduct.

A private university is owned and operated by a religious organization. The university is accredited by the Department of Education of the state in which it is located. This accreditation certifies that the university meets prescribed educational standards. Because it is accredited, the university qualifies for state funding for certain of its operating expenses. Under this funding program, 25 percent of the university's total operating budget comes from state funds. A professor at the university was a part-time columnist for the local newspaper. In one of her published columns, the professor argued that "religion has become a negative force in society." The university subsequently discharged the professor, giving as its sole reason for the dismissal her authorship and publication of this column. The professor sued the university, claiming only that her discharge violated her constitutional right to freedom of speech. The university moved to dismiss the professor's lawsuit on the ground that the U.S. Constitution does not provide the professor with a cause of action in this case.

Should the court grant the university's motion to dismiss?


A. Yes, because the First and Fourteenth Amendments protect the right of the university to employ only individuals who share and communicate its views.B. Yes, because the action of the university in discharging the professor is not attributable to the state for purposes of the Fourteenth Amendment.C. No, because the accreditation and partial funding of the university by the state are sufficient to justify the conclusion that the state was an active participant in the discharge of the professor.D. No, because the U.S. Constitution provides a cause of action against any state-

accredited institution that restricts freedom of speech as a condition of employment.


Incorrect: Answer choice B is correct. The protections afforded by the Due Process Clause of the Fourteenth Amendment apply only to conduct that is attributable to the state. A typical example of a situation not involving state action includes schools that receive government funds but are operated by a private corporation. Because the professor was discharged by a private university and not by a state actor, the Fourteenth Amendment does not apply, and the case should be dismissed. Answer choice A is incorrect because the First and Fourteenth Amendments do not grant a university (specifically when performing an action attributable to the state) any right to discharge employees who do not share and communicate the views of the university. The university's motion to dismiss nevertheless should be granted because the protections afforded by the Fourteenth Amendment apply only to conduct that is attributable to the state. Answer choice C is incorrect because the protections afforded by the Fourteenth Amendment apply only to conduct that is attributable to the state. U.S. Supreme Court precedent establishes that the accreditation, regulation, and partial funding of a university by the state does not rende

Two defendants were being tried together in federal court for bank robbery. The prosecutor sought to introduce testimony from the first defendant's prison cellmate. The cellmate would testify that the first defendant had admitted to the cellmate that he and the second defendant had robbed the bank. The prosecutor asked the court to instruct the jury that the cellmate's testimony could be considered only against the first defendant.
Can the cellmate's testimony be admitted in a joint trial over the second defendant's objection?


A. No, because the first defendant made the statement without Miranda warnings.B. No, because the limiting instruction cannot ensure that the jury will not consider the testimony in its deliberations regarding the second defendant.C. Yes, because the first defendant's statement was a declaration against penal interest.D. Yes, because the limiting instruction sufficiently protects the second defendant.

Answer choice B is correct. The limiting instruction is constitutionally insufficient to avoid the risk that the jury will consider the incriminating statement against the second defendant, who has no opportunity at trial to confront the first defendant. Accordingly, answer choice D is incorrect. Answer choice A is incorrect because Miranda warnings were not required, because the first defendant was not compelled by a known law enforcement agent to make the statement, and in any event the second defendant could not assert Miranda rights belonging to the first defendant. Answer choice C is incorrect because the first defendant's statement incriminating the second defendant could not, under the Sixth Amendment confrontation clause, be considered against the second defendant on a theory that it constitutes a declaration against penal interest.

FBI agents, without a warrant and without permission of Mexican law enforcement or judicial officers, entered Mexico, kidnapped the defendant, an American citizen wanted in the United States for drug smuggling violations, and forcibly drove him back to Texas. Thereafter, the agents, again without a warrant, broke into the Texas home of a woman wanted as a confederate of the defendant, and arrested her. The defendant and his confederate were both indicted for narcotics violations. Both moved to dismiss the indictment on the ground that their arrests violated the Fourth Amendment.

The court should:


A. grant the motions of both the defendant and his confederate.B. grant the motion of the defendant and deny the motion of his confederate.C. grant the motion of the defendant's confederate and deny the motion of the defendant.D. deny the motions of both the defendant and his confederate.

Answer choice D is correct. The police generally do not need a warrant to make a valid arrest in a public place. In situations where a felony has been committed outside the presence of the one making the arrest, a police officer may arrest anyone whom he reasonably believes has committed a felony. The facts state that the defendant and his confederate were "wanted," which implies that the FBI agents had probable cause to believe they had committed felonies. Although the FBI was authorized to arrest the defendant without a warrant, their entry into Mexico and subsequent taking of him was illegal. An unlawful arrest in and of itself, however, has no bearing on a subsequent criminal prosecution. Therefore, the defendant's unlawful arrest is no defense, and his motion should be denied. As for the confederate's arrest, absent exigent circumstances, the police must have a warrant to arrest a person in her own home, which they did not in this case. Again, however, even though the confederate's warrantless arrest was improper, an unlawful arrest alone is no defense in a subsequent criminal prosecution. As long as the police had probable cause, which the facts lead us to believe they did, then the confederate's motion should also be denied. Thus, answer choices A, B, and C are incorrect.

A store owner whose jewelry store had recently been robbed was shown by a police detective a photograph of the defendant, who previously had committed other similar crimes. The store owner examined the photograph and then asked the detective whether the police believed that the man pictured was the robber. After the detective said, "We're pretty sure," the store owner stated that the man in the photograph was the one who had robbed her. The defendant was indicted for the robbery. His counsel moved to suppress any trial testimony by the store owner identifying the defendant as the robber.

Should the court grant the motion and suppress the store owner's trial testimony identifying the defendant as the robber?


A. No, because suppression of in-court testimony is not a proper remedy, even though the out-of-court identification was improper.B. No, because the out-of-court identification was not improper.C. Yes, because the improper out-of-court identification has necessarily tainted any in-court identification.D. Yes, unless the prosecution demonstrates that the in-court identification is reliable.

Answer choice D is correct. Even if an out-of-court identification procedure is unnecessarily suggestive, which this one plainly was, suppression of in-court testimony is not required if the eyewitness's identification is shown to be reliable under a multi-factor inquiry. Answer choice A is incorrect. An improper out-of-court identification procedure may require suppression of in-court testimony if it produces a substantial likelihood of irreparable misidentification. However, suppression of in-court testimony is not required if the eyewitness's identification is shown to be reliable under a multi-factor inquiry. Answer choice B is incorrect. An out-of-court identification procedure is improper if it is unnecessarily suggestive, which this procedure (involving just one photograph and the detective's leading statement) plainly was. Answer choice C is incorrect because, as previously explained, suppression of in-court testimony is not required if the eyewitness's identification is shown to be reliable under a multi-factor inquiry.

A defendant is on trial for evading $100,000 in taxes. The prosecution offers in evidence an anonymous letter to the IRS, identified as being in the defendant's handwriting, saying, "I promised my mother on her deathbed I would try to pay my back taxes. Here is $10,000. I'll make other payments if you promise not to prosecute. Answer yes by personal ad saying, 'OK on tax deal.' "

The letter is


A. admissible, as a statement of present intention or plan.B. admissible, as an admission of a party-opponent.C. inadmissible, because it is an effort to settle a claim.D. inadmissible, because the probative value is substantially outweighed by the risk of unfair prejudice.

Answer choice B is correct. A prior out-of-court statement by a party to the current litigation that is used against that party is not hearsay. The letter is such a statement and is therefore admissible against the defendant. Answer choice A is incorrect because the statement is not hearsay, and so does not fall under a hearsay exception. Answer choice C is incorrect because the $10,000 was given unconditionally, not as an offer to settle the claim. Although the letter suggests that future payments may be made in return for a promise not prosecute, there was no such present offer to compromise. Answer choice D is incorrect because the statement is highly probative and is specifically admissible as an admission.

A state statute requires each insurance company that offers burglary insurance policies in the state to charge a uniform rate for such insurance to all of its customers residing within the same county in that state. So long as it complies with this requirement, a company is free to charge whatever rate the market will bear for its burglary insurance policies. An insurance company located in the state files suit in federal district court against appropriate state officials to challenge this statute on constitutional grounds. The insurance company wishes to charge customers residing within the same county in the state rates for burglary insurance policies that will vary because they would be based on the specific nature of the customer's business, on its precise location, and on its past claims record.

In this suit, the court should


A. Hold the statute unconstitutional, because the statute deprives the insurance company of its liberty or property without due process of law.B. Hold the statute unconstitutional, because the statute imposes an undue burden on interstate commerce.C. Hold the statute constitutional, because the statute is a reasonable exercise of the state's police power.


D. Abstain from ruling on the merit of this case until the state courts have had an opportunity to pass on the constitutionality of this state statute.

Answer choice C is correct. The insurance regulation is valid under the state's traditional police powers. A considerable amount of state regulation is permissible, if there is a valid reason for the regulation and the regulation is as burdensome on local residents as it is on out-of-state commerce. Answer choice A is incorrect because an economic regulation such as this must only pass the minimal rational basis test, which it does. Answer choice B is incorrect because the regulation in this case was made for a valid reason and does not discriminate against out-of-state residents or burden interstate commerce. Answer choice D is incorrect because the facts do not indicate that there are any state law issues that would independently resolve the case; it is dependent upon the constitutionality of the statute.

A state provides by statute, "No person may be awarded any state construction contract without agreeing to employ only citizens of the state and of the United States in performance of the contract."

In evaluating the constitutionality of this state statute under the Supremacy Clause, which of the following would be most directly relevant?


A. The general unemployment rate in the nation.B. The treaties and immigration laws of the United States.C. The need of the state for this particular statute.D. The number of aliens currently residing in the state.

Answer choice B is correct. Congress has sole authority over issues of citizenship, and is said to preempt any state action or law under the Supremacy Clause found in Article VI. (Note that this statute may have other constitutional issues associated with a state prohibition against employment of aliens, but this question only asks about its constitutionality under the Supremacy Clause.) Answer choice A is incorrect because national unemployment rates would not be relevant to a state's preference for its own citizens. Answer choice C is incorrect because if federal law preempted this statute, the state's need would not matter. Answer choice D is incorrect because the number of aliens within the state would not affect the language of either the state or federal laws and would therefore not prevent any preemption.


A prosecutor presented to a federal grand jury the testimony of a witness in order to secure a defendant's indictment for theft of government property. The prosecutor did not disclose to the grand jury that the witness had been convicted four years earlier of perjury. The grand jury returned an indictment, and the defendant pleaded not guilty.
Shortly thereafter, the prosecutor took the case to trial, calling the witness to testify before the jury. The prosecutor did not disclose the witness's prior perjury conviction until the defense was preparing to rest. Defense counsel immediately moved for a mistrial, which the court denied. Instead, the court allowed the defense to recall the witness for the purpose of impeaching him with this conviction, but the witness could not be located. The court then allowed the defense to introduce documentary evidence of the witness's criminal record to the jury before resting its case. The jury convicted the defendant.
The defendant has moved for a new trial, arguing that the prosecutor's failure to disclose the witness's prior conviction in a timely manner violated the defendant's right to due process of law.
If the court grants the defendant's motion, what will be the most likely reason?


A. The defendant was unable to cross-examine the witness about the conviction.B. The prosecutor failed to inform the grand jury of the witness's conviction.C. The court found it reasonably probable that the defendant would have been acquitted had the defense had timely access to the information about the witness's conviction.D. The court found that the prosecutor had deliberately delayed disclosing the witness's conviction to obtain a strategic advantage.

Answer choice C is correct. The untimely disclosure of evidence favorable to the defense (including impeachment information) violates the Constitution if the evidence would have created a reasonable probability of a different outcome had it been disclosed earlier. Answer choice A is incorrect because the court did not limit the defendant's right to cross-examine the witness. Answer choice B is incorrect because the prosecutor is not required to present a grand jury with evidence favorable to a defendant. Answer choice D is incorrect because the prosecutor's motive is not an element of a constitutional claim involving untimely disclosure of evidence favorable to the defense (including impeachment information).


A defendant is tried for armed robbery of a bank. The prosecution, in its case in chief, offers evidence that when the defendant was arrested one day after the crime, he had a quantity of heroin and a hypodermic needle in his possession.

This evidence should be


A. admitted to prove the defendant's motive to commit the crime.B. admitted to prove the defendant's propensity to commit crimes.C. excluded, because its probative value is substantially outweighed by the danger of unfair prejudice.D. excluded, because such evidence may be offered only to rebut evidence of good character offered by defendant.

Answer choice C is correct. While this evidence may expose a motive by the defendant to commit a robbery in order to buy a large amount of drugs, the spillover effect of this relevant evidence would likely mislead and prejudice a trier of fact deciding only the armed robbery charge. Answer choice A is incorrect because, although the evidence could potentially be admissible as circumstantial evidence of motive under the MIMIC rule, the prejudicial effect would substantially outweigh the probative value. Answer choice B is incorrect because evidence of a defendant's bad character is never admissible to prove that the defendant has a propensity to commit crimes and therefore is likely to have committed the crime in question. Answer choice D is incorrect because the possession of a hypodermic needle is most likely not relevant to the defendant's propensity to commit robbery, and answer choice C is a better answer.

A woman and her sister took a trip to the Caribbean. When they passed through U.S. Customs inspection upon their return, the customs officials found liquid cocaine in several bottles each of them was carrying. They were arrested. Upon separate questioning by customs officers, the woman broke down and cried, "I told my sister there were too many officers at this airport." The sister did not give a statement. The woman and her sister were indicted for conspiracy to import cocaine. They were tried separately. At the woman's trial, after the government introduced the above evidence and rested its case, her lawyer moved for a judgment of acquittal on grounds of insufficient evidence.

Should the court grant the motion?


A. No, because the evidence shows that both the woman and her sister agreed to import cocaine.B. No, because the evidence shows that both the woman and her sister possessed cocaine.C. Yes, because the evidence shows only that the woman and her sister committed separate crimes of cocaine possession.D. Yes, because the evidence shows that the woman effectively withdrew from the conspiracy when she cooperated by giving a statement.

Answer choice A is correct. A conviction of conspiracy requires proof of an agreement to commit a crime and, in some jurisdictions or under some statutes, proof of an overt act in furtherance of the agreement. The conspiratorial agreement need not be proven through direct evidence, as long as the circumstantial evidence taken in the light most favorable to the prosecution is sufficient to allow a rational jury to find that there was a conspiratorial agreement beyond a reasonable doubt. Here, because a jury rationally could conclude from all the circumstances (including the woman's statement about her discussion with her sister) that the woman and her sister had agreed to import cocaine, the issue is for the jury, and the woman is not entitled to a court-ordered acquittal. Answer choice B is incorrect because the fact that both the woman and her sister possessed cocaine does not necessarily show that they agreed with each other to do so. Answer choice C is incorrect because a jury rationally could conclude beyond a reasonable doubt from all the circumstances that the woman and her sister had agreed to import cocaine. There is sufficient evidence of conspiracy in this case to send the case to the jury. Answer choice D is incorrect because a defendant cannot effectively withdraw from a conspiracy simply by confessing to the conspiracy after the crime is complete.


The defendant was arrested on a murder charge. She was given Miranda warnings and refused to talk further with the police. At trial, she testified in her own defense. She recounted in some detail her whereabouts on the day of the crime and explained why she could not have committed the crime. On cross-examination and over defense objection, the prosecution emphasized the fact that she did not tell the police this story following her arrest. The prosecution thereby suggested that her testimony was false. The defendant was convicted. On appeal, she claims error in the prosecutor's cross-examination.

Her conviction will most probably be:


A. affirmed, because the defendant's silence at time of arrest is tantamount to a prior inconsistent statement, giving rise to an inference that the story was fabricated.B. affirmed, because the defendant's silence was not used as direct evidence but only for impeachment, a purpose consistent with legitimate cross-examination.C. reversed, because post-arrest silence constituted the defendant's exercise of her Miranda rights and use of that silence against her at trial violated due process.D. reversed, because to require the defense to acquaint the prosecution with the defendant's testimony prior to trial would constitute unconstitutional pre-trial discovery.

Answer choice C is correct. The Fifth Amendment guarantees the right against self-incrimination. In the Miranda warnings, a defendant is told of their right to remain silent, which will not be used against her, which, arguably, could encourage a defendant to remain silent. Any use of silence after arrest and Miranda, whether for substantive or impeachment purposes, is a violation of the defendant's due process rights. Here, the defendant elected not to speak to the police after being advised of her Miranda rights. Prosecutorial comment on her post-arrest silence would be violative of her Fifth Amendment right. Answer choices A and B are incorrect because they ignore the importance of the Fifth Amendment right against self-incrimination and the use of the defendant's silence would render it meaningless. Answer choice D is similarly incorrect because at issue is the defendant's Fifth Amendment right, not a discovery violation.

The defendant was arrested in his girlfriend's apartment after her neighbors had reported sounds of a struggle and the police had arrived to find the defendant bent over his girlfriend's prostrate body. The girlfriend was rushed to the hospital where she lapsed into a coma. Despite the explanation that he was trying to revive his girlfriend after she suddenly collapsed, the defendant was charged with attempted rape and assault after a neighbor informed the police that she had heard the girlfriend sobbing, "No, please no, let me alone."

At trial, the forensic evidence was inconclusive. The jury acquitted the defendant of attempted rape but convicted him of assault. While he was serving his sentence for assault, the girlfriend, who had never recovered from the coma, died. The defendant was then indicted and tried on a charge of felony murder. In this common-law jurisdiction, there is no statute that prevents a prosecutor from proceeding in this manner, but the defendant argued that a second trial for felony murder after his original trial for attempted rape and assault would violate the double jeopardy clause.

His claim is:


A. correct, because he was acquitted of the attempted rape charge.B. correct, because he was convicted of the assault charge.C. incorrect, because the girlfriend had not died at the time of the first trial and he was not placed in jeopardy for murder.D. incorrect, because he was convicted of the assault charge.

Answer choice A is correct. Felony murder applies when an unlawful killing occurs during the course of a felony. In a common law jurisdiction such as this one, rape is a felony, but assault is not. Therefore, only rape may be the basis for a charge of felony murder. Prosecuting the defendant for felony murder would require re-trying the rape charge, which would violate the Fifth Amendment's Double Jeopardy provision prohibiting multiple criminal prosecutions for the same offense. Answer choice B is incorrect although it comes to the right conclusion because it is for the wrong reason: assault is not a felony at common law, so it cannot be the basis for felony murder. Answer choice C is incorrect because, although it is true that the defendant was not placed in jeopardy for murder at the time of his trial, his acquittal on the underlying felony precludes a subsequent prosecution on the same charge. Answer choice D is incorrect because his misdemeanor conviction has no bearing on a potential felony murder charge against him.

A sugar wholesaler contracted to deliver to a candy store owner her sugar requirements for a two year period. Before delivery of the first scheduled installment, the sugar wholesaler sold its business and "assigned" all of its sale contracts to a third party, another reputable and longtime sugar wholesaler. The sugar wholesaler informed the candy store owner of this transaction. When the third party tendered the first installment to the candy store owner in compliance with the contract between the producer and the sugar wholesaler, the candy store owner refused to accept the goods.

In an action by the third party against the candy store owner,


A. the candy store owner will prevail because executory requirements contracts are non-assignable.B. the candy store owner will prevail because duties under an executory bilateral contract are assumable only by an express promise to perform on the part of the delegatee.C. the candy store owner will prevail because language of "assignment" in the transfer for value of a bilateral sale-of-goods contract affects only a transfer of rights, not a delegation of duties.D. The third party will prevail as the contract was properly delegated.

Answer choice D is correct. Generally, obligations under a contract can be delegated, unless the other party to the contract has a substantial interest in having the delegating individual perform (for example, in a personal services contract involving taste or a special skill) or when delegation is prohibited in the contract. Here, there was a proper delegation of the contract by the sugar wholesaler to the third party. There is no indication that the candy store owner required special skill for the sugar or that the delegation was prohibited in the contract. Answer choice A is incorrect, as executory requirements contracts can be assigned. Answer choice B is incorrect, as such duties may be delegated in the absence of an express promise to perform. The delegator remains liable for performance if the delegatee does not perform. Answer choice C is incorrect, as the use of the word "assignment," which does technically apply only to rights, not duties, does not control. The intent of the parties controls, and clearly, the third party accepted delegation of the duty and tried to perform. The foregoing NCBE MBE question has been modified to reflect current NCBE stylistic approaches; the NCBE has not reviewed or endorsed this modification.

A statute provides: A person commits the crime of rape if he has sexual intercourse with a female, not his wife, without her consent.
The defendant is charged with the rape of a woman. At trial, the woman testifies to facts sufficient for a jury to find that the defendant had sexual intercourse with her, that she did not consent, and that the two were not married. The defendant testifies in his own defense that he believed that the woman had consented to sexual intercourse and that she was his common-law wife.

At the conclusion of the case, the court instructed the jury that in order to find the defendant guilty of rape, it must find beyond a reasonable doubt that he had sexual intercourse with the woman without her consent.

The court also instructed the jury that it should find the defendant not guilty if it found either that the woman was the defendant's wife or that the defendant reasonably believed that the woman had consented to the sexual intercourse, but that the burden of persuasion as to these issues was on the defendant.

The jury found the defendant guilty, and the defendant appealed, contending that the court's instructions on the issues of whether the woman was his wife and whether he reasonably believed she had consented violated his constitutional rights.

The defendant's constitutional rights were:


A. violated by the instructions as to both issues.B. violated by the instruction as to whether the woman was his wife, but not violated by the instruction on belief as to consent.C. violated by the instruction on belief as to consent, but not violated by the


instruction as to whether the woman was his wife.D. not violated by either part of the instructions.

Answer choice B is correct. The due process requirement of a fair criminal trial requires that the prosecution prove all the elements of the case beyond a reasonable doubt. The issue of whether the woman was the defendant's wife was an element of the case, which the prosecution carries the burden of proving. The court did violate the defendant's rights, therefore, by instructing the jury that the defendant bore the burden of proving that element. The state is not forbidden, however, from placing the burden of proving an affirmative defense on the defendant. Consent is an affirmative defense to rape, which the defense may be burdened with proving. Therefore, there was no violation of the defendant's rights in the court's instruction regarding consent, making answer choice A incorrect. Answer choice C is incorrect because, as explained above, the instruction as to whether the woman was the defendant's wife is an element of the case which must be proven by the prosecution, while consent is an affirmative defense, which the defendant may have the burden of proving. Answer choice D is incorrect because the instruction as to whether the woman was the defendant's wife did violate the defendant's rights.

A man lived on the second floor of a small convenience store/gas station that he owned. One night he refused to sell the defendant a six-pack of beer after hours, saying he could not violate the state laws. The defendant became enraged and deliberately drove his car into one of the gasoline pumps, severing it from its base. There was an ensuing explosion causing a ball of fire to go from the underground gasoline tank into the building. As a result, the building burned to the ground and the store/gas station owner was killed.

In a common-law jurisdiction, if the defendant is charged with murder and arson, he should be:


A. convicted of both offenses.B. convicted of involuntary manslaughter and acquitted of arson.C. convicted of arson and involuntary manslaughter.D. acquitted of both offenses.

Answer choice A is correct. At common law, arson is the malicious burning of the dwelling of another. The defendant's act of driving his car into a gasoline tank created an obvious likelihood that a burning would ensue, thus satisfying the malice requirement of arson. The question limits the discussion to common law arson, but as the gas station was also the owner's home, even the common law definition of dwelling is satisfied. Murder is an unlawful killing of a human being committed with malice aforethought. The defendant's conduct of driving into the gas tank amounts to a reckless indifference to an unjustifiably high risk to human life (depraved heart), which satisfies the malice requirement. Additionally, arson can be the underlying felony for a felony murder conviction. Answer choices B and C are incorrect because involuntary manslaughter is an unintentional homicide committed with criminal negligence or during an unlawful act. Answer choice D is incorrect because the defendant is guilty of both arson and murder.

The owner of a pizza parlor and one of his chefs consult an attorney about a food-poisoning outbreak resulting in a suit by a litigant against the owner and the chef as joint defendants. The attorney calls his investigator into the conference to make notes of what is said, and those present discuss the facts of the incident and the owner's insurance. The owner thereafter files a cross-claim against the chef for indemnity for any damages obtained by the litigant. The chef calls the investigator in his defense against the cross-claim. He seeks to have the investigator testify to an admission made by the owner in the conference.

On objection by the owner, the court should rule the investigator's testimony


A. admissible, because the attorney-client privilege does not apply, in suits between those conferring with him, to joint consultations with an attorney.B. admissible, because the attorney-client privilege does not apply to testimony by one who does not stand in a confidential relationship with the person against whom the evidence is offered.C. admissible, because the conference was not intended to be confidential, since it concerned anticipated testimony in open court.D. inadmissible, because the owner has not waived the attorney-client privilege.

Answer choice A is correct. Although a confidential communication between attorney and client is generally privileged, the client is the holder of the privilege. Here, when both the owner and the chef sought the advice of the lawyer, either may seek to reveal the privileged information in a cross-claim between themselves. Therefore, answer choice D is incorrect. Answer choice B is incorrect because there is no such exception to the attorney-client privilege. Answer choice C is incorrect because the matter was privately discussed for the purposes of legal advice and not made in public. Here, the conference remained confidential because the investigator is a representative of the attorney and the chef is a representative of the owner, so privilege is not destroyed. The foregoing NCBE MBE question has been modified to reflect current NCBE stylistic approaches; the NCBE has not reviewed or endorsed this modification.


content-neutral restrictions on speech must be:

narrowly tailored to a legitimate interest and leave open alternate channels of communication

An owner owns his home, Blackacre, which was mortgaged to a bank by a duly recorded purchase money mortgage. Last year, the owner replaced all of Blackacre's old windows with new windows. Each new window consists of a window frame with three inserts: regular windows, storm windows, and screens. The windows are designed so that each insert can be easily inserted or removed from the window frame without tools to adjust to seasonal change and to facilitate the cleaning of the inserts. The new windows were expensive. The owner purchased them on credit, signed a financing statement, and granted a security interest in the windows to the supplier of the windows. The supplier promptly and properly filed and recorded the financing statement before the windows were installed. The owner stored the old windows in the basement of Blackacre. This year, the owner has suffered severe financial reverses and has defaulted on his mortgage obligation to the bank and on his obligation to the supplier of the windows. The bank brought an appropriate action to enjoin the supplier from its proposed repossession of the window inserts.

In the action, the court should rule for


A. the bank, because its mortgage was recorded first.B. the bank, because windows and screens, no matter their characteristics, are an integral part of a house.C. the supplier, because the inserts are removable.D. the supplier, because the availability of the old windows enables the bank to return Blackacre to its original condition.

Answer choice C is correct. U.C.C. Article 9 gives a holder of a security interest in fixtures the right to remove the fixtures after default, as long as the fixtures are removable. Because the facts indicate the windows are removable, the supplier is entitled to remove them. Answer choice A is incorrect because Article 9 gives priority to a purchase money security interest in removable fixtures even over a prior recorded mortgage, but only as to those fixtures. The supplier has a purchase money security interest in the windows because was given in connection with the acquisition of the windows. Answer choices B and D are incorrect because the key issue is whether the windows are removable, which is clearly specified in the facts.


A defendant was prosecuted for armed robbery. At trial, the defendant testified in his own behalf, denying that he had committed the robbery. On cross-examination, the prosecutor intends to ask the defendant whether he had been convicted of burglary six years earlier.

The question concerning the burglary conviction is


A. proper if the court finds that the probative value for impeachment outweighs the prejudice to the defendant.B. proper, because the prosecutor is entitled to make this inquiry as a matter of right.C. improper, because burglary does not involve dishonesty or false statement.D. improper, because the conviction must be proved by court record, not by question on cross-examination.

Answer choice A is correct. A criminal defendant has no obligation to testify under his right against self-incrimination. If he does, however, he is subject to cross-examination and impeachment, which can include questions about his past crimes, under certain circumstances. If the crime involved dishonesty or false statement, a witness may always be impeached with evidence that the witness was convicted of the crime within 10 years of the conviction. If the crime does not involve dishonesty or false statement, the court has discretion to bar impeachment evidence, if the witness is a criminal defendant and the prosecution failed to show that the probative value outweighs the prejudicial effect. Burglary, theft, and robbery are generally not considered crimes involving dishonesty or false statement. Thus, the prosecutor is not entitled to make the inquiry as of right and must show that the probative value outweighs the prejudice to the accused. Thus, answer choice B is incorrect. Answer choice C is incorrect because even though burglary does not involve dishonesty or false statement, other felonies may be used for impeachment purposes, subject to a probative-prejudicial balancing test. Answer choice D is incorrect because evidence of a prior conviction may be produced by way of an admission of the witness, whether during direct testimony or on cross-examination, as well as by extrinsic evidence.


On May 15, a brother and a sister signed a writing in which the brother, "in consideration of $500 to be paid to the brother by the sister," offered the sister the right to purchase property for $250,000 within 30 days. The writing further provided, "This offer will become effective as an option only if and when the $500 consideration is in fact paid." On May 30, the brother, having received no payment or other communication from the sister, sold and conveyed property to a lawyer for $270,000. On May 31, the brother received a letter from the sister enclosing a cashier's check for $500 payable to the brother and stating, "I am hereby exercising my option to purchase property and am prepared to close whenever you're ready."

Which of the following, if proved, best supports the sister's suit against the brother for breach of contract?


A. The sister was unaware of the sale to the lawyer when the owner received the letter and check from the buyer on May 31.B. On May 30, the sister decided to purchase the property and applied for and obtained a commitment from a bank for a $225,000 loan to help finance the purchase.C. When the May 15 writing was signed, the brother said to the sister, "Don't worry about the $500; the recital of '$500 to be paid' makes this deal binding."D. The brother and the sister are both professional dealers in real estate.

Answer choice A is correct. The mailbox rule, which makes acceptance effective upon dispatch, does not apply if the parties specify some other point of acceptance, as is the case here (where acceptance is effective upon receipt of the $500). The offer was revocable until the $500 was paid, but because the revocation is effective upon receipt, and the brother did not give proper notice to the sister of the sale to the lawyer, the offer was not revoked in time, and an option contract was created. Answer choice B is incorrect because reliance does not act as an acceptance, and the brother was free to revoke the offer; he just failed to do so. Answer choice C is incorrect because the parol evidence rule would bar a contemporaneous or prior inconsistent statement, such as the one in answer choice C. Answer choice D is incorrect because the brother's and the sister's professions are simply not relevant. The foregoing NCBE MBE question has been modified to reflect current NCBE stylistic approaches; the NCBE has not reviewed or endorsed this modification.

A manufacturer sells shoes with the manufacturer's brand name to two competing retailers. When the larger of the two retailers, a national retailer, learns that the manufacturer is also selling the same brand of shoes to its competitor, the retailer stops placing orders for the shoes with the manufacturer. In response to a query by the manufacturer as to the reason for larger retailer's action, the large retailer states that it does not want to carry the same brand name shoes as the smaller regional retailer. Under no contractual obligation to sell the brand name shoes to the regional retailer, the manufacturer stops doing so. The national retailer resumes ordering the brand name shoes from the manufacturer. The regional retailer, while suffering a loss of income due to its inability to sell the brand name shoes, continued to be profitable since shoe sales represented only a small fraction of its overall sales. In an action for intentional interference with a contract brought by the regional retailer against the national retailer, which of the following would be the national retailer's weakest argument?


A. There was no contract between the manufacturer and the regional retailer.B. The national retailer did not substantially interfere with the regional retailer's overall business.C. The regional retailer was a competitor.D. The national retailer is merely exercising its freedom to refuse to deal with another.


Incorrect: Answer choi

Answer choice B is correct. To prove intentional interference with a contract, the regional retailer must prove that the national retailer (1) knew of a contractual relationship between the regional retailer and the manufacturer, (2) intentionally interfered with the contract, which resulted in a breach, and (3) that the breach caused damages to the regional retailer. The regional retailer need only show a pecuniary loss of the benefits from the contract; there is no requirement that the interference prevent the regional retailer from being profitable or that the loss be substantial. Therefore, the argument that the national retailer did not substantially interfere is a weak argument, and answer choice B is correct. Answer choice A is incorrect because when the interference involves a prospective economic relationship, rather than an existing contractual relationship, the defendant has greater latitude in acting in its own interest, particularly if the plaintiff is a competitor. Therefore, answer choice A would be a strong argument for the national retailer to make. Answer choice C is also a strong argument and therefore incorrect because the relationship between the parties is a factor in determining whether the interference is improper. Generally, a competitor has greater latitude in acting in a manner that interferes with the business relationships of a competitor. Answer choice D is incorrect because a person is generally free to deal or to refuse to deal with another. Thus, this argument is stronger than answer choice B.

A developer was the owner of a large subdivision. A purchaser became interested in purchasing a lot but could not decide between Lot 40 and Lot 41. The price and fair market value of each of those two lots was $5,000. The purchaser paid the developer $5,000, which the developer accepted, and the developer delivered to the purchaser a deed which was properly executed, complete, and ready for recording in every detail except that the space in the deed for the lot number was left blank. The developer told the purchaser to fill in either Lot 40 or Lot 41 according to his decision and then to record the deed. The purchaser visited the development the next day and completely changed his mind, selecting Lot 25. He filled in Lot 25 and duly recorded the deed. The price of Lot 25 and its fair market value was $7,500. Before the developer had time to learn of the purchaser's actions, the purchaser sold Lot 25 to another buyer for $6,000 by a duly and properly executed, delivered, and recorded warranty deed. The buyer knew that the developer had put a price of $7,500 on Lot 25, but he knew no other facts regarding the developer-purchaser transaction. The buyer's attorney accurately reported the purchaser's record title to be good, marketable, and free of encumbrances. Neither the buyer nor his attorney made any further investigation outside the record. The developer brought an appropriate action against the second buyer to recover title to Lot 25.

If the developer loses, the most likely basis for the judgment is that


A. the Statute of Frauds prevents the introduction of any evidence of the developer's and the purchaser's agreement.B. recording of the deed from the developer to the purchaser precludes any question of its genuineness.C. as between the developer and a bona fide purchaser, the developer is


estopped.D. the clean hands doctrine bars the developer from relief.

Answer choice C is correct. A grantor who allows the grantee to obtain physical possession of the deed may be estopped from asserting that the delivery was invalid against a bona fide purchaser from the grantee. Since the developer negligently permitted the purchaser to have possession of the deed, he is estopped from recovering title. Answer choice A is incorrect because it is the parol evidence rule, not the Statute of Frauds, which prevents the introduction of any evidence of an outside agreement. Answer choice B is incorrect because the recording of a deed is not an absolute guaranty of its genuineness, as this option states. Answer choice D is incorrect because the clean hands doctrine applies to actions made in bad faith, which is not the case here - the developer behaved negligently, but there is no evidence that he wasn't acting in good faith.

A defendant was charged with attempted murder. At the preliminary hearing, the presiding judge heard the testimony of four prosecution witnesses and found that the prosecution had failed to establish probable cause that the defendant had committed any offense. Accordingly, he dismissed the charge. The prosecutor then called the same four witnesses before a grand jury. The grand jury indicted the same defendant for attempted murder. The defendant has moved to quash the indictment on the ground of double jeopardy.

How should the court proceed?


A. Grant the motion, because the dismissal of the first charge on the merits, whether correct or incorrect, bars any further prosecution.B. Grant the motion, unless the prosecution has evidence that was not presented in the first case.C. Deny the motion, because the defendant has not yet been in jeopardy of conviction on the attempted murder charge.D. Deny the motion, because the protection of the double jeopardy clause does not come into play until there has been a conviction or an acquittal.

Answer choice C is correct. For double jeopardy purposes, jeopardy does not attach until trial, when the jury is sworn in (or, in a bench trial, when the first witness is sworn in). Answer choices A and B are incorrect because the dismissal after a preliminary hearing had no double jeopardy consequences for the reasons stated above. Answer choice D is incorrect. While jeopardy does not attach until trial, actions short of a conviction or acquittal (for example, a mistrial) may bar retrial on double jeopardy grounds.

A pedestrian died from injuries caused when a driver’s car struck him. The executor of the pedestrian’s estate sued the driver for wrongful death. At trial, the executor calls a nurse to testify that two days after the accident, the pedestrian said to the nurse, “The car that hit me ran the red light.” Fifteen minutes thereafter, the pedestrian died.
As a foundation for introducing evidence of the pedestrian’s statement, the executor offers to the court a doctor’s affidavit that the doctor was the intern on duty the day of the pedestrian’s death and that several times that day the pedestrian had said that he knew he was about to die.
Is the affidavit properly considered by the court in ruling on the admissibility of the pedestrian’s statement?


A. No, because it is hearsay not within any exception.B. No, because it is irrelevant since dying declarations cannot be used except in prosecutions for homicide.C. Yes, because, though hearsay, it is a statement of then-existing mental condition.D. Yes, because the judge may consider hearsay in ruling on preliminary questions.

Answer choice D is correct. Rule 104(a) provides that when deciding preliminary questions of law, including the admissibility of evidence, the trial judge is not bound by the rules of evidence, except those with respect to privilege. Deciding whether the pedestrian’s statement is admissible is a preliminary question of law for the judge (as opposed to a question of fact for the jury), and the judge may consider any unprivileged evidence, including hearsay, in the course of making that decision. Therefore, answer choice A is incorrect. Answer choice B is incorrect not only for the above reason, but also because it is an untrue statement; dying declarations may be used in both civil and criminal trials. Answer choice C is incorrect because the state-of-mind hearsay exception applies only to descriptions of a then-existing mental condition (such as intent, plan, or motive) being offered to prove the condition in question or conduct in conformity with the expressed intent. The pedestrian’s statement about the car running the red light does not fall into this category.

A defendant was tried for armed robbery. The state introduced evidence that a man, identified by witnesses as the defendant, entered a convenience store at 11 p.m. on March 5, threatened the clerk with a gun, and took $75 from the cash register. The defendant did not testify, but his sister did. She testified that on March 5, at the time of the robbery, the defendant was with her in a city 300 miles away. On cross-examination, the sister admitted having given a statement to the police in which she had said that the defendant was not with her on March 5, but she claimed that the earlier statement was mistaken. The court instructed the jury that in order to convict the defendant, they had to find all of the elements of the crime beyond a reasonable doubt.

As to the defendant's claim of alibi, which of the following additional instructions would be proper?


A. Alibi is a matter of defense and so must be established by the defendant; however, the burden of persuasion is by a preponderance of the evidence, not beyond a reasonable doubt.B. Before you may consider the defendant's claim of alibi, you must decide whether he has produced sufficient evidence to raise the issue.C. If you have a reasonable doubt as to whether the defendant was present at the convenience store at about 11 p.m. on March 5, you must find him not guilty.D. If the defendant's evidence has caused you to have a reasonable doubt as to whether he was the robber, you must find him not guilty.

Answer choice C is correct. Due process requires the prosecution to prove all elements of a crime beyond a reasonable doubt. Because alibi is not a traditional defense, but rather negates an essential element of the crime (the defendant's actual commission thereof), due process precludes imposing upon a defendant the burden of proving alibi. For these reasons, answer choices A and B are incorrect. Answer choice D is incorrect. The error in this response is its reference to a reasonable doubt arising from the defendant's evidence. Due process requires the prosecution to prove all elements of a crime beyond a reasonable doubt.


Two friends planned to incorporate a business together and agreed that they would own all of the corporation's stock in equal proportion.
A businesswoman conveyed land by a warranty deed to "the corporation and its successors and assigns." The deed was recorded.
Thereafter, the friends had a disagreement. No papers were ever filed to incorporate the business. There is no applicable statute.
Who owns the land?


A. The businesswoman, because the deed was a warranty deed.B. The businesswoman, because the deed was void.C. The two friends as tenants in common, because they intended to own the corporation's stock in equal proportion.D. The two friends as tenants in common, because they were the intended sole shareholders.

Answer choice B is correct. To be valid, a deed must be properly executed and delivered. A deed to a nonexistent grantee, such as a corporation that has not yet been legally formed, is void. At the time the businesswoman attempted to convey the land to the corporation, the corporation had not yet been legally formed, so the deed was void. Answer choice A is incorrect. The businesswoman owns the land, but she does so because the deed was void. Answer choice C is incorrect because a deed to a nonexistent grantee, such as a corporation that has not yet been legally formed, is void and thus conveys no title. It is irrelevant that the two friends intended to own the corporation's stock in equal proportion. Answer choice D is incorrect because it is irrelevant that the two friends intended to be the sole shareholders. At the time the businesswoman attempted to convey the land to the corporation, the corporation had not yet been legally formed, so the deed was void.


A man and woman were jointly in possession of Greenacre in fee simple as tenants in common. They joined in a mortgage of Greenacre to a bank. The man erected a fence along what he considered to be the true boundary between Greenacre and the adjoining property, owned by a neighbor. Shortly thereafter, the man had an argument with the woman and gave up his possession to Greenacre. The debt secured by the mortgage had not been paid. The neighbor surveyed his land and found that the fence erected a year earlier by the man did not follow the true boundary. Although part of the fence was within Greenacre, part of the fence encroached on the neighbor's land. The neighbor and the woman executed an agreement fixing the boundary line in accordance with the fence constructed by the man. The agreement, which met all the formalities required in the jurisdiction, was promptly and properly recorded. A year after the agreement was recorded, the man temporarily reconciled his differences with the woman and resumed joint possession of Greenacre. Thereafter, the man repudiated the boundary line agreement and brought an appropriate action against the neighbor and the woman to quiet title along the original true boundary.

In such action, the man will


A. win, because the bank was not a party to the agreement.B. win, because one tenant in common cannot bind another tenant in common to a boundary-line agreement.C. lose, because the agreement, as a matter of law, was mutually beneficial to the woman and the man.D. lose, because the woman was in sole possession of said premises at the time the agreement was signed.

Answer choice B is correct. Each cotenant must agree to bind the property to a boundary agreement. Neither the woman nor the neighbor can bind the man with the woman's signature, regardless of whether the bank was a party, or the extent of the benefit to the property. Answer choice A is incorrect because the bank was not a necessary party. Answer choice C is incorrect because it is a misstatement of the law. Answer choice D is incorrect because the woman's sole possession of Greenacre did nothing to diminish the man's interest.


A construction company was digging a trench for a new sewer line in a street in a high-crime neighborhood. During the course of the construction, there had been many thefts of tools and equipment from the construction area. One night, the construction company's employees neglected to place warning lights around the trench. A delivery truck drove into the trench and broke an axle. While the delivery driver was looking for a telephone to summon a tow truck, thieves broke into the delivery truck and stole $350,000 worth of goods. The delivery company sued the construction company to recover for the $350,000 loss and for $1,500 worth of damage to its truck. The construction company stipulated that it was negligent in failing to place warning lights around the trench, and admits liability for damage to the truck, but denies liability for the loss of the goods.
On cross-motions for summary judgment, how should the court rule?


A. Deny both motions, because there is evidence to support a finding that the construction company should have realized that its negligence could create an opportunity for a third party to commit a crime.B. Grant the construction company's motion, because no one could have foreseen that the failure to place warning lights could result in the loss of a cargo of valuable goods.C. Grant the construction company's motion, because the criminal acts of third persons were a superseding cause of the loss.D. Grant the delivery company's motion, because but for the construction company's actions, the goods would not have been stolen.

Answer choice A is the correct answer because a negligent tortfeasor is not generally liable for the criminal acts of third parties made possible by his negligence, but there is an exception when the tortfeasor should have realized the likelihood of the crime at the time of his negligence. The issue of foreseeability is generally a question for the jury. In this case, there had been many thefts from the construction area during the course of construction. Accordingly, there was enough evidence to support a jury verdict for the plaintiff, but it was not so overwhelming as to require the judge to take the rare step of granting summary judgment for the plaintiff. Answer choice B is incorrect because the jury should be asked to consider whether the failure to place warning lights could foreseeably create a situation in which a damaged vehicle would be left vulnerable to theft. Answer choice C is incorrect as there is enough evidence to support a jury finding that the theft was foreseeable, and the defendant's motion for summary judgment should be denied. Answer choice D is wrong as even a negligent tortfeasor is not liable for the criminal acts of third parties made possible by his negligence unless he should have realized the likelihood of such a crime at the time of his negligence. In this case, there remains a jury question as to whether the pattern of past thefts from the construction site made the theft of the goods from the delivery truck foreseeable.

The owner of a milk container manufacturing firm sought to focus public attention on the milk packaging law of his state in order to have it repealed. On a weekday at 12:00 noon, he delivered an excited, animated, and loud harangue on the steps of the State Capitol in front of the main entryway. An audience of 200 onlookers, who gathered on the steps, heckled him and laughed as he delivered his tirade. He repeatedly stated, gesturing expressively and making faces, that "the g-ddamned milk packaging law is stupid," and that "I will strangle every one of those g-ddamned legislators I can get hold of because this law they created proves they are all too dumb to live." After about fifteen minutes, he stopped speaking, and the amused crowd dispersed. A state statute prohibits "all speech-making, picketing, and public gatherings of every sort on the Capitol steps in front of the main entryway between 7:45 a.m.-8:15 a.m., 11:45 a.m.-12:15 p.m., 12:45 p.m.-1:15 p.m., and 4:45 p.m.-5:15 p.m. on Capitol working days."

If the owner is prosecuted under the "Capitol steps" statute and defends on constitutional grounds, which of the following best describes the proper burden of proof?


A. He would have to prove that the state did not have a rational basis for enacting this statute.B. He would have to prove that the state did not have a compelling need for this statute or that it had less restrictive means by which it could satisfy that need.C. The state would have to prove that it had a rational basis for enacting this statute.D. The state would have to prove that it had a compelling need for this statute and that there were no less restrictive means by which it could satisfy that need.

Answer choice D is correct. Applicable to the states via the Fourteenth Amendment, the First Amendment generally prohibits the government's ability to restrict speech. However, the government may regulate the time, place, and manner of speech in a public forum (which can be traditional or designated) so long as the law is (i) content neutral, (ii) narrowly tailored to serve a significant government interest, and (iii) leaves open alternate channels of communication for the speaker. Answer choice D most closely describes the correct burden of proof.

A suspect entered the police station and announced that he wanted to confess to a murder. The police advised the suspect of the Miranda warnings, and he signed a written waiver. The suspect described the murder in detail and pinpointed the location where a murder victim had been found a few weeks before. Later, a court-appointed psychiatrist determined that the suspect was suffering from a serious mental illness that interfered with his ability to make rational choices and to understand his rights and that the psychosis had induced his confession.

The suspect's confession is:


A. admissible, because there was no coercive police conduct in obtaining the suspect's statement.B. admissible, because the suspect was not in custody.C. inadmissible, because the suspect's confession was a product of his mental illness and was therefore involuntary.D. inadmissible, because under these circumstances, there was no valid waiver of Miranda warnings.

Answer choice A is correct. A confession must be voluntary to be admissible. The suspect's confession was voluntary, and the police did not coerce him into making it. Answer choice B is incorrect because the suspect's being in custody does not impact the voluntariness of his statement under these facts. Answer choice C is incorrect because the defendant's mental state alone does not negate the voluntariness of a statement. In this case, there was no coercive conduct by the police, and the police were not the cause of the suspect's mental illness, so the confession was voluntary. Answer choice D is incorrect because the Miranda waiver is irrelevant, since the suspect was not in custody and was not interrogated.

In the plaintiff's antitrust suit against manufacturers of insulation, the plaintiff's interrogatories asked for information concerning total sales of insulation by each of the defendant manufacturers in a particular year. The defendants replied to the interrogatories by referring the plaintiff to the Insulation Manufacturer's Annual Journal for the information.

If, at trial, the plaintiff offers the annual as evidence of the sales volume, this evidence is


A. admissible as an adoptive admission of the defendants.B. admissible as a business record.C. inadmissible, because it is hearsay, not within any exception.D. inadmissible as lacking sufficient authentication.

Answer choice A is correct. Answers to interrogatories, which are signed and made under oath by a party to the litigation are non-hearsay admissions. Answer choice B is incorrect because it ignores the fact that the statement is a party admission, regardless of the form of the evidence. Answer choice C is incorrect because a statement by an opposing party is, by definition, non-hearsay. Answer choice D is incorrect because authentication of the journal is irrelevant as to the adoptive admission.

On November 1, a plumber and a professional basketball player contracted for the sale by the plumber to the professional basketball player of the collection of antique basketballs that the plumber had inherited from his grandfather. The professional basketball player agreed to pay the purchase price of $22,000 when the plumber delivered the antique basketballs on February 19. On February 1, the professional basketball player received a signed letter from the plumber that stated: "I have decided to dispose of the cases for the antique basketballs that you have already purchased. If you want the cases, I will deliver them to you along with the basketballs on February 19 at no additional cost to you. Let me know before February 15 whether you want them. I will not sell them to anyone else before then." On February 14, the professional basketball player e-mailed and the plumber received the following message: "I accept your offer of the cases." The plumber was not a merchant with respect to either antique basketballs or cases.

The plumber is contractually obligated to deliver the cases because


A. The professional basketball player provided a new bargained-for exchange by agreeing to take the cases.B. The plumber's letter (received by the professional basketball player on February 1) and the professional basketball player's e-mail message of February 14 constituted an effective modification of the original sale-of-basketballs contract.C. The professional basketball player's e-mail message of February 14 operated to rescind unilaterally the original sale-of-basketballs contract.D. The plumber's letter (received by the professional basketball player on February 1) waived the bargained-for consideration that would otherwise be required.

Answer choice B is correct. At common law, modification of an existing contract must be supported by consideration, but under the U.C.C., only good faith is required to modify a contract for the sale of goods. Here, there was good faith by both sides, so the modification was effective without consideration. Answer choice A is incorrect, as the professional basketball player did not provide a new bargained-for exchange by agreeing to take the cases. There was no legal detriment to the professional basketball player, but the U.C.C. only requires good faith, not consideration, to modify a contract for the sale of goods. Answer choice C is incorrect, as the e-mail message did not unilaterally rescind the original contract. Answer choice D is incorrect, as bargained-for consideration is not required under the U.C.C. to modify a contract for the sale of goods. The foregoing NCBE MBE question has been modified to reflect current NCBE stylistic approaches; the NCBE has not reviewed or endorsed this modification.

The defendant, a young doctor, is charged with falsely claiming deductions on her federal income tax return. At trial, a witness testified for the defendant that she has a reputation in the community for complete honesty. After a sidebar conference at which the prosecutor gave the judge a record showing that the defendant's medical school had disciplined her for altering her transcript, the prosecutor proposes to ask the witness on cross-examination: "Have you ever heard that the defendant falsified her medical school transcript?"

Is the prosecutor's question proper?


A. No, because it calls for hearsay not within any exception.B. No, because its minimal relevance on the issue of income tax fraud is substantially outweighed by the danger of unfair prejudice.C. Yes, because an affirmative answer will be probative of the defendant's bad character for honesty and, therefore, her guilt.D. Yes, because an affirmative answer will impeach the witness's credibility.

Answer choice D is the correct answer because the incident can be offered on cross-examination of the character witness, the proper purpose being to show that the witness's assessment of the defendant's character for honesty is not credible. The intent of the question is to test the witness's knowledge of the defendant's reputation on the one hand, and the quality of the community on the other. If the witness hasn't heard about the falsification, he might not be very plugged in to the community and so might be a poor reputation witness. On the other hand, if the witness answers "yes," then the jury might infer that the community in which the defendant has a reputation for complete honesty may be setting the honesty bar too low. In either case, the alleged falsification is probative impeachment whether or not it occurred. Note that the courts require that the cross-examiner must have a good faith belief that the event actually occurred before inquiring into the act on cross-examination. In this case, that good faith standard is met by the evidence presented at the sidebar conference that the defendant was disciplined in medical school. Answer choice A is incorrect because the question does not call for a statement that would be used for its truth. Therefore, it is not hearsay. Answer choice B is incorrect because the alleged incident is not offered to prove income tax fraud. Answer choice C is incorrect because Federal Rule of Evidence 405 prohibits evidence of specific acts indicative of a person's character when that character evidence is offered to prove that a person acted in accordance with the character trait on the occasion in question at trial. Thus, the prosecutor may not introduce the incident involving the medical school transcript for the inference that, because the defendant acted dishonestly on that occasion, she likely acted dishonestly with regard to her tax return.

A plaintiff brought an action against a defendant for injuries received in an automobile accident, alleging the defendant's negligent behavior in speeding and driving inattentively. The plaintiff calls a witness to testify that the defendant had a reputation in the community for being a reckless driver and was known as "dare-devil Dan."

The witness's testimony is


A. admissible as habit evidence.B. admissible, because it tends to prove that the defendant was negligent at the time of this collision.C. inadmissible, because the defendant has not offered testimony of his own good character.D. inadmissible to show negligence.

Answer choice D is correct. In civil cases, evidence of a person's character is generally inadmissible as irrelevant to prove conduct in conformity with that character trait. Here, the witness's testimony is being offered to show that the defendant has a reputation for being a negligent driver. Because this kind of character evidence is inadmissible, answer choice B is incorrect. Evidence of a person's habit is admissible to prove conduct in conformity with the habit on a particular occasion. A habit is a person's particular routine reaction to a specific set of circumstances. Answer choice A is incorrect because the testimony offered does not qualify as habit evidence. Answer choice C is incorrect because it states the rule regarding character evidence in criminal cases and this is a civil case.

A defendant is being tried for the murder of a woman who disappeared 10 years ago and has not been heard from since. Her body has never been found. The prosecutor has presented strong circumstantial evidence that she was murdered by the defendant. To help establish the fact of her death, the prosecutor has requested that the judge give the following instruction, based on a recognized presumption in the jurisdiction: "A person missing and not heard from in the last seven years shall be presumed to be deceased."

Is the instruction proper?


A. No, because the fact that someone has not been heard from in seven years does not necessarily lead to a conclusion that the person is dead.B. No, because mandatory presumptions are not allowed against a criminal defendant on an element of the charged crime.C. Yes, because it expresses a rational conclusion that the jury should be required to accept.D. Yes, because the defendant has a chance to rebut the presumption by offering evidence that the woman is alive or has been heard from in the last seven years.

Answer choice B is correct. The U.S. Supreme Court has held it to be a violation of due process for a judge to give a mandatory jury instruction in a criminal case on an element of the charged crime. The instruction is unconstitutional because the phrase "shall be presumed" could be interpreted by the jury as shifting the burden of proof to the defendant or as requiring the jury to find an element of the charged crime, neither of which is permissible. Answer choice A is incorrect. It is true that the conclusion does not necessarily follow. But to be constitutional, a presumption or inference does not have to be certain; it needs only to be rational and to follow more likely than not. The presumption in the requested instruction meets that standard. However, the instruction violates due process as explained above. Answer choice C is incorrect because a mandatory jury instruction in a criminal case on an element of the charged crime violates due process. Answer choice D is incorrect. It is true that the defendant has a chance to rebut the presumption with evidence that the woman is alive or has been heard from in the last seven years. But it violates due process to shift the burden of proof to the defendant on an element of the crime.

A man suffered a broken jaw in a fight with a neighbor that took place when they were both spectators at a soccer match.
If the man sues the neighbor for personal injury damages, which of the following actions must the trial court take if requested by the man?


A. Prevent the neighbor's principal eyewitness from testifying, upon a showing that six years ago the witness was convicted of perjury and the conviction has not been the subject of a pardon or annulment.B. Refuse to let the neighbor cross-examine the man's medical expert on matters not covered on direct examination of the expert.C. Exclude nonparty eyewitnesses from the courtroom during the testimony of other witnesses.D. Require the production of a writing used before trial to refresh a witness's memory.

Answer choice C is correct. If a party moves to exclude prospective witnesses before they testify, "the court must order witnesses excluded so they cannot hear other witnesses' testimony." Answer choice A is incorrect because a witness can never be excluded from testifying simply because there is impeachment evidence that could be used against that witness. All witnesses are presumed to be competent. The man can use this evidence to impeach the witness when the witness testifies. Answer choice B is incorrect because the trial court is not required to prohibit a cross-examiner from asking questions unrelated to the direct examination. Rule 611(b) states that the court "may allow inquiry into additional matters." Answer choice D is incorrect because the trial court has discretion to order a party to produce for the adversary a writing used before trial to refresh the memory of a witness called by the party, but the court is not required to do so.

The aged mother of a sister and a brother, both adults, wished to employ a live-in companion so that she might continue to live in her own home. The mother, however, had only enough income to pay one-half of the companion's $2,000 monthly salary. Learning of their mother's plight, the sister and brother agreed with each other in a signed writing that, on the last day of January and each succeeding month during their mother's lifetime, each would give the mother $500. The mother then hired the companion. The sister and brother made the agreed payments in January, February, and March. In April, however, the brother refused to make any payment and notified the sister and the mother that he would make no further payments. Assume that there is a valid contract between the sister and brother and that the mother has declined to sue the brother.

Will the sister succeed in an action against the brother in which she asks the court to order him to continue to make his payments to the mother under the terms of their contract?


A. Yes, because the sister's remedy at law is inadequate.B. Yes, because the sister's burden of supporting her mother will be increased if the brother does not contribute his share.C. No, because a court will not grant specific performance of a promise to pay money.D. No, because the brother's breach of contract has caused no economic harm to the sister.

Answer choice A is correct. The sister here is seeking an order of specific performance against the brother. When damages are an inadequate remedy, the non-breaching party may pursue the equitable remedy of specific performance. Here, it is unclear how long the mother may live, so damages do not seem to be an adequate remedy. An order requiring the brother to pay his share for as long as the mother lives would be an adequate remedy. Answer choice B is incorrect, as the fact that the sister's burden will be increased is not what controls the remedy. Rather, specific performance is appropriate here because damages are not an adequate remedy. Answer choice C is incorrect, as courts do grant specific performance of promises to pay money where damages at law would be inadequate. Answer choice D is not correct, as it would be likely that the burden of supporting the mother would fall on the sister in the absence of payment by the brother. There was a contract between the brother and sister and damages at law would be inadequate.

The plaintiff, a jockey, was seriously injured in a race when another jockey, the defendant, cut too sharply in front of her without adequate clearance. The two horses collided, causing the plaintiff to fall to the ground, sustaining injury. The State Racetrack Commission ruled that, by cutting in too sharply, the defendant committed a foul in violation of racetrack rules requiring adequate clearance for crossing lanes. The plaintiff has brought an action against the defendant for damages in which one count is based on battery. Will the plaintiff prevail on the battery claim?


A. Yes, if the defendant was reckless in cutting across in front of the plaintiff's horse.B. Yes, because the State Racetrack Commission determined that the defendant committed a foul in violation of rules applicable to racing.C. No, unless the defendant intended to cause impermissible contact between the two horses or apprehension of such contact by the plaintiff.D. No, because the plaintiff assumed the risk of accidental injury inherent in riding as a jockey in a horse race.

Answer choice C is correct. One is liable for battery when she intentionally engages in a harmful or offensive contact with another without the other’s consent. Battery is an intentional tort. If the defendant did not intend to make contact between the two horses or intend to frighten the plaintiff, she cannot be liable. Answer choice A is incorrect because mere recklessness is insufficient to establish liability in a battery action. The defendant must have intended the contact. Answer choice B is incorrect. If the Commission found the defendant in violation of a racing rule, she may be liable for negligence per se in a negligence action if that is the type of harm to be protected. The plaintiff, however, has filed a claim for battery. Answer choice D is incorrect because assumption of the risk is not a defense to an intentional tort.


A nurse worked as a private duty nurse and on occasion worked in the hospital. The hospital called a private duty referral agency through which the nurse usually obtained employment, and asked that in the future she not be assigned to patients in the hospital. The agency asked the hospital why it had made the request. The hospital sent a letter to the agency giving as the reason for its request that significant amounts of narcotics had disappeared during the nurse's shift from the nursing stations at which she had worked.
If the nurse asserts a claim based on defamation against the hospital, the nurse will


A. Recover, because the hospital accused the nurse of improper professional conduct.B. Recover if the nurse did not take the narcotics.C. Not recover if narcotics disappeared during the nurse's shifts.D. Not recover if the hospital reasonably believed that the nurse took the narcotics.

Answer choice D is correct. Defamation is the publication of information disseminated by the defendant to a third party that is about the plaintiff and injures the plaintiff’s reputation. A defamatory statement must be false, and if the plaintiff is a private figure, the defendant must have published the statement at least negligently. Further, statements made in the interest of the recipient are subject to a conditional privilege, so long as the defendant did not act with malice. Here, there is nothing in the facts to indicate that the hospital acted negligently, so the nurse will not prevail, making answer choices A and B incorrect. Even if the hospital were negligent, however, it has the available defense of qualified privilege because the statements made by the hospital were in the interest of the recipient (the agency). There is no evidence that the statements were made with malice. And, if the hospital reasonably believed that the statement was true, as indicated by answer choice D, then there is no evidence to conclude the hospital acted with malice. Answer choice A is incorrect because although some statements made about improper professional conduct may constitute defamation, recovery is not absolute. There are several defenses, including truth, available to the defendant. Answer choice B is incorrect because even if the nurse did not take the narcotics, she would still have to prove that the hospital was negligent as to the truth or falsity of its statement. Answer choice C is incorrect because it does not address whether the hospital believed the statements it was making were true. Answer choice D is a better option than answer choice C because D eliminates the possibility that the hospital acted with malice, which is a possibility under C (maybe the hospital knew that someone else had taken them but blamed her anyway). The possibility of malice means it’s possible that the privilege wouldn’t apply; under D’s scenario, the privilege is more likely to be applicable.

The legislature of a state is debating reforms in the law governing insanity. Two reforms have been proposed. Proposal A would eliminate the insanity defense altogether. Proposal B would retain the defense but place on the defendant the burden of proving insanity by a preponderance of the evidence. Opponents of the reforms argue that the proposals would be unconstitutional under the Due Process Clause of the United States Constitution.

Which of the proposed reforms would be unconstitutional?


A. Both proposals.B. Neither proposal.C. Proposal A only.D. Proposal B only.

Answer choice B is correct. There is no due process right to present a defense of insanity, or any other particular affirmative defense, so Proposal A is not unconstitutional. Under Proposal B, the defendant is burdened with proving an insanity defense. The Supreme Court has held that the Due Process Clause does not forbid placing the burden of proving an affirmative defense, such as insanity or self-defense, on the defendant. Therefore, Proposal B is not unconstitutional. Answer choices A, C, and D are incorrect because, as discussed above, neither of the two proposals is unconstitutional.

A defendant is charged with mail fraud. At trial, the defendant has not taken the witness stand, but he has called a witness who has testified that the defendant has a reputation for honesty. On cross-examination, the prosecutor seeks to ask the witness, "Didn't you hear that two years ago the defendant was arrested for embezzlement?"
Should the court permit the question?


A. No, because the defendant has not testified and therefore has not put his character at issue.B. No, because the incident was an arrest, not a conviction.C. Yes, because it seeks to impeach the credibility of the witness.D. Yes, because the earlier arrest for a crime of dishonesty makes the defendant's guilt of the mail fraud more likely.

Answer choice C is correct. The witness has testified that she knows about the defendant's reputation. The prosecutor has the right to test the basis and adequacy of that knowledge, as well as the nature of the community itself. If the witness answers that she had not heard about the arrest, that admission could indicate that she is not very knowledgeable about the defendant's reputation in the community, because such an arrest would likely have a negative effect on that reputation. If the witness says that she had heard about the arrest, a negative inference could be raised about the community itself and its view of what it is to be an honest person. Answer choice A is incorrect because when a defendant calls a character witness, the prosecutor is permitted to test the character witness's knowledge of the defendant. The fact that the defendant has not put his character at issue is irrelevant. Answer choice B is incorrect because for purposes of testing the witness's knowledge of the defendant's reputation for honesty, the bad act need not have resulted in a conviction. An arrest is sufficient to have an impact on the community'

state enacted a statute to regulate administratively the conduct of motor vehicle junkyard businesses in order to deter motor vehicle theft and trafficking in stolen motor vehicles or parts thereof. The statute requires a junkyard owner or operator to permit representatives of the Department of Motor Vehicles or of any law enforcement agency upon request during normal business hours to take physical inventory of motor vehicles and parts thereof on the premises. The statute also states that a failure to comply with any of its requirements constitutes a felony. Police officers assigned to the city's Automobile Crimes Unit periodically visited all motor vehicle junkyards in town to make the inspections permitted by the statute. A woman owned such a business in the city. One summer day, the officers asked to inspect the vehicles on her lot. The woman said, "Do I have a choice?" The officers told her she did not. The officers conducted their inspection and discovered three stolen automobiles. The woman is charged with receiving stolen property. The woman moves pretrial to suppress the evidence relating to the three automobiles on the ground that the inspection was unconstitutional.

Her motion should be:


A. sustained, because the statute grants unbridled discretion to law enforcement officers to make warrantless searches.B. sustained, because the stated regulatory purpose of the statute is a pretext to circumvent the warrant requirement in conducting criminal investigations.C. denied, because the statute deals reasonably with a highly regulated industry.D. denied, because administrative searches of commercial establishments do not require warrants.

Answer choice C is correct. The Supreme Court has held that warrantless searches of businesses in highly regulated industries, including automobile junkyards, are permissible. Therefore, the statute in question and the inspections that it authorizes are not unconstitutional, and the evidence is admissible. Answer choice A is incorrect because it is untrue; the statute does not give law enforcement unbridled discretion, as they are required to make requests and the request must be made during normal business hours. Answer choice B is incorrect because a specific exception to the warrant requirement exists permitting searches of automobile junkyards; therefore, the stated purpose of the regulation is legitimate. Answer choice D is incorrect because it is an untrue statement; there is no general warrant exception for commercial establishments.

A state law that restricted abortion was challenged in state court as a violation of the Due Process Clause of the Fourteenth Amendment to the U.S. Constitution and as a violation of a similar due process provision of the state constitution. The case made its way to the state's highest court, which ruled that the law violated the due process provisions of both the U.S. and the state constitutions.

If petitioned to do so, may the U.S. Supreme Court exercise jurisdiction to review the state court decision?


A. No, because the state court's decision in this case rests on adequate and independent state law grounds.B. No, because the U.S. Supreme Court has appellate jurisdiction only over state court decisions that determine the constitutionality of federal laws.C. Yes, because the U.S. Supreme Court has appellate jurisdiction over any ruling of a state's highest court based on an interpretation of federal law.D. Yes, because the U.S. Supreme Court has appellate jurisdiction over decisions that find state laws in violation of the federal Constitution.

Answer choice A is correct. The U.S. Supreme Court does not have appellate jurisdiction over a decision by the highest court of a state when that decision is supported by state law grounds that are (1) independent of federal law and (2) adequate to sustain the result in the case. The result in this case is fully supported by the state court's ruling that the law violated the state constitution, and this ruling is independent of any consideration of the federal constitutional claim. Accordingly, even if the U.S. Supreme Court were to reverse the state court's ruling on the federal constitutional issue, the result in the case would not change, and thus the decision is not reviewable. Answer choice B is incorrect because while this response correctly states that the U.S. Supreme Court may not review the decision, it inaccurately states the scope of the Court's appellate jurisdiction. Answer choice C is incorrect because the U.S. Supreme Court may not review this decision. While the state court opinion did cite a violation of federal law, it also stated that the law violated the due process clause of the state constitution. The opinion therefore reflects sufficiently adequate and independent state grounds preventing the U.S. Supreme Court from reviewing the case. Answer choice D is incorrect because while the statement in this response is generally true, the U.S. Supreme Court does not have appellate jurisdiction over a decision of the highest court of a state finding a state law in violation of the federal Constitution when that decision is supported by state law grounds that are (1) independent of federal law and (2) adequate to sustain the result in the case.

The defendant, a real estate developer, was trying to purchase land on which he intended to build a large commercial development. The plaintiff, an elderly widow, had rejected all of the defendant's offers to buy her ancestral home, where she had lived all her life and which was located in the middle of the defendant's planned development. Finally, the defendant offered her $250,000. He told her that it was his last offer and that if she rejected it, state law authorized him to have her property condemned. The plaintiff then consulted her nephew, a law student, who researched the question and advised her that the defendant had no power of condemnation under state law. The plaintiff had been badly frightened by the defendant's threat, and was outraged when she learned that the defendant had lied to her. If the plaintiff sues the defendant for damages for emotional distress, will she prevail?


A. Yes, if the defendant's action was extreme and outrageous.B. Yes, because the plaintiff was frightened and outraged.C. No, if the plaintiff did not suffer emotional distress that was severe.D. No, if it was not the defendant's purpose to cause emotional distress.

Answer choice C is correct. Intentional infliction of emotional distress is the use of extreme and outrageous conduct to intentionally or recklessly inflict severe emotional distress. In order to prevail on her claim, the plaintiff must satisfy all of the requisite elements. Here, the plaintiff did not suffer severe emotional distress resulting from the defendant's actions. Therefore, she cannot prevail on her claim. Answer choice A is incorrect because even if the defendant's actions were extreme and outrageous, the plaintiff cannot recover unless she suffered severe emotional distress. Answer choice B is incorrect as it is not the appropriate legal standard. Answer choice D is incorrect because even if the defendant did not intend to cause emotional distress, he will be liable if his actions were reckless under the circumstances.

A male defendant is tried for aggravated assault and battery of a female jogger. At the request of police, the jogger prepared a sketch bearing a strong likeness to the defendant, but she died in an automobile accident before the defendant was arrested.

At trial, the prosecution offers the sketch. The sketch is


A. admissible as an identification of a person after perceiving him.B. admissible as past recollection recorded.C. inadmissible as hearsay not within any exception.D. inadmissible as an opinion of the jogger.

Answer choice C is correct. Because the sketch is being introduced by a witness other than the jogger, it is hearsay: an out-of-court statement introduced to prove the truth of the matter asserted. No hearsay exception exists to allow the admission of the sketch. Answer choice A is incorrect because the sketch does not qualify as a present sense impression made while the jogger was perceiving the event. Answer choice B is incorrect because such evidence is admissible only if (1) the witness is available; (2) the witness states that she has insufficient recollection of the event; and (3) she is given a chance to consult the writing and verify it as her own. Here, the jogger is unavailable because she died before trial. Answer choice D is incorrect because, without the jogger present to testify, the sketch is inadmissible hearsay, not opinion testimony. The foregoing NCBE MBE question has been modified to reflect current NCBE stylistic approaches; the NCBE has not reviewed or endorsed this modification.


A rancher and a trainer of horses signed the following writing: "For $5,000, the rancher will sell to the trainer a gray horse that the trainer may choose from among the grays on the rancher's ranch." The trainer refused to accept delivery of a gray horse timely tendered by the rancher or to choose among those remaining, on the ground that, during their negotiations, the rancher had orally agreed to include a saddle, worth $100, and also to give the trainer the option to choose a gray or a brown horse. The trainer insisted on one of the rancher's brown horses, but the rancher refused to part with any of his browns or with the saddle as demanded by the trainer.

If the trainer sues the rancher for damages and seeks to introduce evidence of the alleged oral agreement, the court probably willA. admit the evidence as to both the saddle and the option to choose a brown horse.B. admit the evidence as to the saddle but not the option to choose a brown horse.C. admit the evidence as to the option to choose a brown horse but not the promise to include the saddle.D. not admit any of the evidence

Answer choice B is correct. Under the parol evidence rule, evidence that contradicts the writing is generally inadmissible, but evidence that supplements a contract that is partially integrated is admissible if it is consistent with the writing and does not contradict the terms. Here, the evidence about the saddle supplements the written agreement and does not contradict it. The evidence about the option to choose a brown horse contradicts the written agreement. Answer choice A is incorrect, as the evidence about the option to choose a brown horse contradicts the written agreement and would be kept out by the parol evidence rule. Answer choice C is incorrect, as the evidence about the option to choose a brown horse contradicts the written agreement. Answer choice D is incorrect, as the evidence about the saddle merely supplements the written agreement and does not contradict it and therefore would be admissible under the parol evidence rule.

A defendant is charged with aggravated assault on a victim, a game warden. The defendant testified that, when he was confronted by the victim, who was armed and out of uniform, the defendant believed the victim was a robber and shot in self-defense. The state calls a witness to testify that a year earlier, he had seen the defendant shoot a man without provocation and thereafter falsely claim self-defense.

The witness's testimony is


A. admissible, as evidence of the defendant's untruthfulness.B. admissible, as evidence that the defendant did not act in self-defense on this occasion.C. inadmissible, because it is improper character evidence.D. inadmissible, because it is irrelevant to the defense the defendant raised.

Answer choice C is correct. Character evidence is any document or testimony offered to prove that a person acted in a particular way on a particular occasion, based on that person's disposition or character. A defendant's past crimes or wrongs may be admissible as MIMIC (Motive, Intent, absence of Mistake, Identity, or Common plan or scheme) evidence, but are not admissible to show criminal propensity. The witness's testimony does not satisfy the MIMIC rule, and is improper character evidence being offered to prove that the defendant has a propensity to commit a particular crime based on past action. Answer choice A is incorrect because the defendant's untruthfulness may be established only by impeachment on cross-examination; it may not be established by extrinsic evidence. Answer choice B is incorrect because the evidence is not probative of the defendant's actions on the specific occasion in question, but is highly prejudicial. Answer choice D is incorrect because the testimony is somewhat relevant; however, its probative value is substantially outweighed by the danger of unfair prejudice, making answer choice C a better answer.


The defendant is tried on a charge of driving while intoxicated. When the defendant was booked at the police station, a videotape was made that showed him unsteady, abusive, and speaking in a slurred manner.

If the prosecutor lays a foundation properly identifying the tape, should the court admit it in evidence and permit it to be shown to the jury?


A. Yes, because it is an admission.B. Yes, because its value is not substantially outweighed by unfair prejudice.C. No, because the privilege against self-incrimination is applicable.D. No, because specific instances of conduct cannot be proved by extrinsic evidence.

Answer choice B is correct. The Fifth Amendment right against self-incrimination applies only to statements made by the defendant and not actions. Real and objective evidence taken from the defendant, such as the videotape here, are not protected. Since the videotape, and statements made therein, are not testimonial in nature (and not the product of a custodial interrogation), Miranda is inapplicable. Answer choice A in incorrect as the videotape would not constitute an admission and the defendant's non-testimonial behavior is not protected by any constitutional right. Here, answer choice C is incorrect because routine booking questions are not normally considered "interrogation" under a Fifth Amendment analysis. Additionally, the defendant's behavior is not testimonial in nature and is not constitutionally protected. Answer choice D is incorrect because such evidence is admissible, though a jury may consider its weight.

On March 31, a bakery and a department store entered into a written agreement in which the bakery agreed to bake and sell to the department store 5,000 custom-designed extra large cookies for the store to sell in its gourmet foods section. The contract provided that the department store would pay half of the purchase price on May 15 in order to give the bakery capital to "tool up" for the work; that the bakery would deliver 2,500 cookies on May 31; that the department store would pay the balance of the purchase price on June 15; and that the bakery would deliver the balance of the cookies on June 30. On May 10, the bakery notified the department store that it was doubtful whether the bakery could perform because of problems encountered in modifying its production machines to produce the cookies. On May 15, however, the bakery assured the department store that the production difficulties had been overcome, and the department store paid the bakery the first 50 percent installment of the purchase price. The bakery did not deliver the first 2,500 cookies on May 31, or at any time thereafter; and on June 10, the bakery notified the department store that it would not perform the contract.


commercially reasonable time, but if the store awaits performance beyond that period, it cannot recover any resulting damages that it reasonably could have avoided.D. The department store has a cause of action for breach of contract that it can successfully assert only after it has given the bakery a commercially reasonable time to perform.

Which of the following is NOT a correct statement of the parties' legal status immediately after the bakery's notice on June 10?


A. The department store has a cause of action for total breach of contract because of the bakery's repudiation, but that cause of action will be lost if the bakery retracts its repudiation before the store changes it position or manifests to the bakery that it considers the repudiation final.B. The department store can bring suit to rescind the contract even if it elects to await the bakery's performance for a commercially reasonable time.C. The department store can await performance by the bakery for a

Answer choice D is correct. Under the UCC, the department store has several alternative remedies if the bakery fails to tender the goods. The store may treat the repudiation as a breach unless the bakery retracts the repudiation prior to the store changing its position in reliance on the repudiation or acknowledging the repudiation as a breach; hence, answer choice A is not the correct answer (note that the question asks for the remedy that is NOT available). Answer choice B is not the correct answer because, under the UCC, the department store may wait a commercially reasonable period of time before suing to rescind the contract. Likewise, answer choice C is not correct because, under the UCC, waiting beyond a commercially reasonable time will not defeat the department store's claim, but will be treated as a failure to mitigate additional damages. Only answer choice D is not provided for in the UCC; once notified of the breach, the department store may, but is not required, to give the bakery a "commercially reasonable time to perform."

In a written contract, an architect agreed to draw up the plans for and to supervise construction of a client's new house. In return, the client agreed to pay the architect a fee of $10,000 to be paid upon the house's completion. After completion, the client claimed erroneously but in good faith that the architect's plans were defective. The client orally offered to pay the architect $7,500 in full settlement of the claim for the fee. The architect orally accepted that offer despite the fact that the reasonable value of his services was in fact $10,000. The client paid the architect $7,500 pursuant to their agreement. The architect subsequently sued the client for the remaining $2,500. In a preliminary finding, the trier of fact found that there were no defects in the architect's plans.

Will the architect be likely to prevail in his action against the client for $2,500?


A. Yes, because payment of $7,500 cannot furnish consideration for the architect's promise to surrender his claim.B. Yes, because the oral agreement to modify the written contract is not enforceable.C. No, because the architect's promise to accept $7,500 became binding when the client made the payment.D. No, because the architect's acceptance of partial payment constituted a novation.

Answer choice C is correct. The architect's agreement to accept a payment for less than the amount due constituted an effective accord, supported by consideration, which was satisfied with the payment of $7,500. Consideration is present because of the good faith dispute as to the amount owed. By compromising, each party surrenders its respective claim as to how much is owed. Answer choice A is incorrect as the rule of Foakes v. Beer, which is a specific application of the pre-existing duty rule, provides that part payment of a liquidated debt is invalid for a lack of consideration. The rule does not apply, however, where there is a compromise of a claim disputed in good faith. This exception applies even if it later becomes apparent that the reason for disputing the claim was invalid. Answer choice B is incorrect because the Statute of Frauds is removed as a defense when an oral agreement is fully performed. Moreover, the dispositive issue is whether consideration exists to enforce the agreement to settle the claim. Answer choice D is not the correct answer because a novation arises where either one or both of the parties to a contract is replaced by a third party. The architect's acceptance of payment does not constitute a novation since neither the client nor the architect was replaced by a third party.

A state entered into a contract with a construction company for construction of a four-lane turnpike. Prior to commencement of construction, the legislature, in order to provide funds for parks, repealed the statute authorizing the turnpike and canceled the agreement with the company. The company sued the state to enforce its original agreement.

In ruling on this case, a court should hold that the state statute cancelling the agreement is


A. Valid, because constitutionally the sovereign is not liable except with its own consent.B. Valid, because the legislature is vested with constitutional authority to repeal laws it has enacted.C. Invalid, because a state is equitably estopped to disclaim a valid bid once accepted by it.D. Invalid, because of the constitutional prohibition against impairment of contracts.

Answer choice D is correct. Article I, Section 10 of the U.S. Constitution prohibits states from passing any law "impairing the obligation of contracts." This prohibition applies only to state legislation that retroactively impairs contractual rights. Here, the state legislation will retroactively impair a public contract for a reason that is not "reasonable and necessary." The state will likely not be able to demonstrate an important interest that cannot be served by less restrictive means, and there has been no unforeseen circumstance that makes the cancellation of this contract necessary. Answer choice A is incorrect because of the prohibition retroactively impairing contractual rights. Answer choice B is incorrect because of the prohibition retroactively impairing contractual rights. Answer choice C is incorrect because there would be no substantial impairment if the state had reserved the right to revoke, alter or amend such a contract.

A defendant is being prosecuted for conspiracy to possess with intent to distribute cocaine. At trial, the government seeks to have its agent testify to a conversation that he overheard between the defendant and a co-conspirator regarding the incoming shipment of a large quantity of cocaine. That conversation was also audiotaped, though critical portions of it are inaudible. The defendant objects to the testimony of the agent on the ground that it is not the best evidence of the conversation.

Is the testimony admissible?


A. No, because the testimony of the agent is not the best evidence of the conversation.B. No, because the testimony of the agent is hearsay not within any exception.C. Yes, because the best evidence rule does not require proof of the conversation through the audiotape.D. Yes, because the audiotape is partly inaudible.

Answer choice C is correct. Since the agent personally heard the conversation, he is not relying on the contents of the audiotape for his testimony. Therefore there is no violation of the best evidence rule. The agent is not attempting to prove the contents of the audiotape, only what he overheard. The conversation could, of course, also be proved by introducing the audiotape, and it can be argued that the audiotape is "better" evidence than the agent's testimony. Nonetheless, so long as the agent obtained his knowledge of the conversation from his own perception, and not by listening to the audiotape, the best evidence rule is not violated. Answer choice A is incorrect because, so long as the agent obtained his knowledge of the conversation from his own perception, and not by listening to the audiotape, the best evidence rule is not violated. Answer choice B is incorrect. The statements made by the defendant are admissible as individual admissions under Federal Rule of Evidence 801(d)(2)(A). The statements made by the co-conspirator are admissible as admissions of a co-conspirator under Federal Rule of Evidence 801(d)(2)(E). Answer choice D is incorrect. The testimony of the agent is admissible because it is based on the personal perception of the agent who overheard the conversation. Therefore, the agent will be allowed to testify to what he personally heard regardless of whether any recording was made and regardless of any defects in the recording.


A lawn service company agreed in writing to purchase from a supplier all of its requirements for lawn care products during the next calendar year. In the writing, the supplier agreed to fulfill those requirements and to give the company a 10% discount off its published prices, but it reserved the right to increase the published prices during the year. After the parties had performed under the agreement for three months, the supplier notified the company that it would no longer give the company the 10% discount off the published prices.

Does the company have a viable claim against the supplier for breach of contract?


A. Yes, because part performance of the agreement by both parties made it enforceable for the full year.B. Yes, because the company's agreement to buy all of its lawn care products from the supplier made the agreement enforceable.C. No, because the supplier could, and did, revoke its offer with respect to future deliveries.D. No, because the absence of a minimum quantity term rendered the company's promise illusory.

Answer choice B is correct. The parties entered into an enforceable requirements contract under UCC § 2-306. Although the terms of the parties' agreement granted the supplier the discretion to increase the published prices during the year, the contract did not grant the supplier the right to discontinue the promised 10% discount off the published prices. The supplier's refusal to give the company the 10% discount was a breach of the contract's agreed-upon terms. Answer choice A is incorrect because, pursuant to the terms of their contract, the parties agreed to a one-year term. Therefore, part performance is not required in order for the contract to be enforceable for a full year. Answer choice C is incorrect because revocation relates to the contract formation process. Given that mutual assent has occurred, revocation is not a relevant issue. Answer choice D is incorrect. UCC § 2-306(1) validates the enforceability of requirements contracts even if the parties do not include a minimum quantity term in their agreement. Therefore, the absence of a minimum quantity term in the parties' requirements contract did not render it illusory, and the contract is enforceable.


The defendant became intoxicated at a bar. He got into his car and drove away. Within a few blocks, craving another drink, he stopped his car in the middle of the street, picked up a brick, and broke the display window of a liquor store. As he was reaching for a bottle, the night watchman arrived. Startled, the defendant turned and struck the watchman on the head with the bottle, killing him. Relevant statutes define burglary to include "breaking and entering a building not used as a dwelling with the intent to commit a crime therein." Murder is defined as "the premeditated and intentional killing of another or the killing of another in the commission of committing rape, robbery, burglary, or arson." Another statute provides that intoxication is not a defense to a crime unless it negates an element of the offense. The defendant was charged with the murder of the watchman.

At the defendant's trial, the court should in substance charge the jury on the issue of the defense of intoxication that:


A. intoxication is a defense to the underlying crime of burglary if the defendant, due to drunkenness, did not form an intent to commit a crime within the building, in which case there can be no conviction for murder unless the defendant intentionally and with premeditation killed the watchman.B. voluntary intoxication is not a defense to the crime of murder.C. the defendant is guilty of murder despite his intoxication only if the state proves beyond a reasonable doubt that the killing of the watchman was premeditated and intentional.D. voluntary intoxication is a defense to the crime of murder if the defendant would not have killed the watchman but for his intoxication.

Answer choice A is correct. Answer choice A is the most complete statement of the statutory law given in the fact pattern as it deals with the felony murder portion of the definition of murder. Here, intoxication can negate the element of burglary, which requires intent to commit a crime in the building, and thus it prevents a conviction under the felony murder portion of the given statute. So the defendant can only be found guilty of murder if he intentionally and with premeditation killed the watchman. Answer choice B is incorrect and not a complete statement of law because voluntary intoxication may prevent the formation of premeditation and the mental state of intent, both required in the statute provided. Answer choice C is incorrect because the language "only if" precludes the defendant from being found guilty of felony murder. Answer choice D is an incorrect interpretation of the statute provided and also precludes a conviction under the felony murder portion of the statute. The foregoing NCBE MBE question has been modified to reflect current NCBE stylistic approaches; the NCBE has not reviewed or endorsed this modification.


A construction company sued a developer for money owed on a cost-plus contract that required notice of proposed expenditures beyond original estimates. The defendant asserted that it never received the required notice. At trial, the plaintiff calls its general manager to testify that it is the plaintiff’s routine practice to send cost overrun notices as required by the contract. The general manager also offers a photocopy of the cost overrun notice letter to the defendant on which the plaintiff is relying, and which he has taken from the plaintiff’s regular business files.
On the issue of giving notice, is the letter copy admissible?


A. Yes, though hearsay, under the business record exception.B. Yes, because of the routine practices of the company.C. No, because it is hearsay not within any exception.D. No, because it is not the best evidence of the notice.

Answer choice B is correct. Rule 406 permits evidence of an organization’s regular response to a specific set of circumstances to be admissible as habit evidence to prove conduct in conformity with the habit on a particular occasion. The letter is being offered as evidence of the plaintiff’s routine business practice, and so is admissible. Answer choices A and C are incorrect because the letter is not hearsay. It is not being offered to prove the truth of its contents (that expenditures exceeded the estimate), but to show that the plaintiff gave the defendant the required notice. Answer choice D is incorrect because the best evidence rule applies only when the contents of a writing are at issue; the plaintiff is not attempting to prove the contents of the document, so the rule does not apply.
The foregoing NCBE MBE question has been modified to reflect current NCBE stylistic approaches; the NCBE has not reviewed or endorsed this modification.

Police officers received a tip that drug dealing was occurring at a certain ground-floor duplex apartment. They decided to stake out the apartment. The stakeout revealed that a significant number of people visited the apartment for short periods of time and then left. A man exited the apartment and started to walk briskly away. The officers grabbed the man and, when he struggled, wrestled him to the ground. They searched him and found a bag of heroin in one of his pockets. After discovering the heroin on the man, the police decided to enter the apartment. They knocked on the door, which was opened by the woman who lived there. The police asked if they could come inside, and the woman gave them permission to do so. Once inside, the officers observed several bags of heroin on the living room table. The woman is charged with possession of the heroin found on the living room table. She moves pretrial to suppress the heroin on the ground that it was obtained by virtue of an illegal search and seizure.

Should the woman's motion be granted?


A. No, because the tip together with the heroin found in the man's pocket provided probable cause for the search.B. No, because the woman consented to the officers' entry.C. Yes, because the officers' decision to enter the house was the fruit of an illegal search of the man.D. Yes, because the officers did not inform the woman that she could refuse consent.

Answer choice B is correct. The woman's consent justified the officers' entry, and the heroin was properly seized because it was in plain view. See, e.g., Illinois v. Rodriguez, 497 U.S. 177 (1990). Answer choice A is incorrect because, although it correctly states that the woman's motion to suppress the heroin should not be granted, it misstates the legal basis for this conclusion. Even assuming there was probable cause to search the home, a warrant would have been required for entry had the woman not consented. See, e.g., Payton v. New York, 445 U.S. 573 (1980). Answer choice C is incorrect because the search of the man, even assuming it was improper, did not violate the woman's rights and therefore provides no basis for suppressing evidence found in her house. Cf. Minnesota v. Carter, 525 U.S. 83 (1998) (discussing what is commonly referred to as "standing" requirement). Answer D is incorrect since there is no requirement that officers inform individuals of their right to refuse consent. See Schneckloth v. Bustamonte, 412 U.S. 218 (1973).

n a city, a number of armed bank robberies were committed near closing time by a masked man wearing a white hooded sweatshirt and blue sweatpants. Police saw a man wearing a white hooded sweatshirt and blue sweatpants pacing nervously outside one of the city's banks just before it closed. The police stopped the man and frisked the outer layers of his clothing for weapons, but found none. They asked the man what he was doing outside the bank and pointed out that he was wearing clothing similar to clothing worn by the perpetrator of recent robberies. After pausing for several moments, the man confessed. The police had not provided him with any Miranda warnings. After being charged with the bank robberies, the man moved to suppress his confession. The parties agreed, and the court properly found, that the police had reasonable suspicion but not probable cause at all times before the man confessed.

Should the man's motion to suppress be granted?


A. Yes, because the confession was the fruit of a Fourth Amendment violation, even though there was no Miranda violation.B. Yes, because the confession was the fruit of a Miranda violation, even though there was no Fourth Amendment violation.C. Yes, because the confession was the fruit of both a Fourth Amendment violation and a Miranda violation.D. No, because there was neither a Fourth Amendment violation nor a Miranda violation.

Answer choice D is correct. There was neither a Fourth Amendment violation nor a Miranda violation. There was no Fourth Amendment violation because the stop, frisk, and questioning were permissible, under Terry v. Ohio, 392 U.S. 1 (1968), based on reasonable suspicion. There was no Miranda violation because warnings are not required for Terry stops. See Berkemer v. McCarty, 468 U.S. 420, 439-40 (1984). Answer choice A is incorrect because, as explained above, there was no Fourth Amendment violation. Answer choice B is incorrect because, as explained above, there was no Fifth Amendment violation. Answer choice C is incorrect for the reasons listed above.

There are two identical twins. The first twin, who is angry at the defendant, said, "You'd better stay out of my way. The next time I find you around here, I'll beat you up." Two days later, while in the neighborhood, the defendant saw the second twin coming toward him. As the second twin came up to the defendant, the second twin raised his hand. Thinking the second twin was the first twin and fearing bodily harm, the defendant struck the second twin.
If the second twin asserts a claim against the defendant and the defendant relies on the privilege of self-defense, the defendant will


A. Not prevail, because the second twin was not an aggressor.B. Not prevail unless the second twin intended his gesture as a threat.C. Prevail if the defendant honestly believed that the second twin would attack him.D. Prevail only if a reasonable person under the circumstances would have believed that the second twin would attack him.

Answer choice D is correct. A person may use reasonable force to defend against an offensive contact or bodily harm that he reasonably believes is about to be intentionally inflicted upon him. A person’s mistaken belief that he is in danger, so long as it is a reasonable mistake, which is likely based on these facts, does not invalidate the defense. Answer choice A is incorrect because the second twin need not be an aggressor in order for the defendant to prevail. The defendant must only reasonably believe that he is about to suffer a battery. Here, the defendant was "fearing bodily harm." Answer choice B is incorrect because the intent of the plaintiff is irrelevant. Answer choice C is incorrect as it states the incorrect legal standard. The defendant's belief must be reasonable, not honest.


A farmer who wanted to sell her land received a letter from a developer that stated, "I will pay you $1,100 an acre for your land." The farmer's letter of reply stated, "I accept your offer." Unbeknownst to the farmer, the developer had intended to offer only $1,000 per acre but had mistakenly typed "$1,100." As both parties knew, comparable land in the vicinity had been selling at prices between $1,000 and $1,200 per acre.
Which of the following states the probable legal consequences of the correspondence between the parties?


A. There is no contract, because the parties attached materially different meanings to the price term.B. There is no enforceable contract, because the developer is entitled to rescission due to a mutual mistake as to a basic assumption of the contract.C. There is a contract formed at a price of $1,000 per acre.D. There is a contract formed at a price of $1,100 per acre.

Answer choice D is correct. An enforceable contract requires mutual assent as determined by the parties' objective, rather than subjective, manifestations of assent. Given the parties' knowledge of the price of comparable land, the developer's offer created a reasonable understanding that the developer would purchase the land for $1,100 per acre. Moreover, because the farmer neither knew nor had reason to know that the developer intended to purchase the land for only $1,000 per acre, the developer will be bound to purchase it for $1,100 per acre. Accordingly, the parties' conduct gave rise to a contract formed at $1,100 per acre when the farmer accepted the developer's offer. For that reason, answer choice C is incorrect. Answer choice A is incorrect because the general rule that contract formation may be defeated, under some circumstances, if parties attach materially different meanings to a material term is inapplicable here; the critical issue relates to the developer's intent, as manifested by his conduct, and the impact of the farmer's lack of knowledge of the developer's mistake. Answer choice B is incorrect because, while a mutual mistake may give rise to an action for rescission, there was no mutual mistake in this case. The critical issue here relates to the developer's intent as manifested by his conduct, and the impact of the farmer's lack of knowledge of the developer's mistake.


During 2009, a rash of vandalism occurred in the wealthiest neighborhood of a city. In early 2010, the city council adopted this resolution: "The city will pay $1,000 for the arrest and conviction of anyone guilty of any of the 2009 vandalism committed here." The foregoing was telecast by one of the city's television stations once daily for one week. Thereafter, in August 2010, the city council by resolution repealed its reward offer and caused this resolution to be broadcast once daily for a week over two local radio stations. In September 2010, following the detective’s investigation, a high school student voluntarily confessed to a private detective to having committed all of the 2009 vandalism. The detective's cousin suggested that the detective claim the city's reward, of which the detective had been previously unaware. The detective immediately made the claim. In December 2010, as a result of the detective's investigation, the student was convicted of the vandalism. The city, which has no immunity to suit, has since refused to pay the detective anything, although he swears that he never heard of the city's repeal before claiming his reward.

In a suit by the detective against the city to recover the $1,000 reward, which of the following, in light of the facts given, most usefully supports the detective's claim?


A. The city was benefited as a result of the detective's services.B. The city's offer was in the nature of a bounty, so that the elements of contract are not essential to the city's liability.C. The fact that the city attempted to revoke its offer only a few months after making it demonstrated that the attempted revocation was in bad faith.D. Although there was no bargained-for exchange between the detective and the

Answer choice B is correct. The detective's strongest argument is that the city's offer was a set bounty, not subject to the normal contract limitations requiring adequate consideration. As a unilateral contract, all the detective had to do is perform as requested. Answer choice A is incorrect because it either accepts that a contract existed or that an equitable remedy for quasi-contract is available. Answer choices C and D are similarly incorrect because they embrace contract concepts not applicable if the offer was a bounty. The foregoing NCBE MBE question has been modified to reflect current NCBE stylistic approaches; the NCBE has not reviewed or endorsed this modification.

n a suit attacking the validity of a deed executed fifteen years ago, a plaintiff alleges mental incompetency of the grantor and offers in evidence a properly authenticated affidavit of the grantor's brother. The affidavit, which was executed shortly after the deed, stated that the grantor's brother had observed the grantor closely over a period of weeks, that the grantor had engaged in instances of unusual behavior (which were described), and that the grantor's appearance had changed from one of neatness and alertness to one of disorder and absentmindedness.

The judge should rule the grantor's brother's affidavit


A. inadmissible as opinion.B. inadmissible as hearsay, not within any exception.C. admissible as an official document.D. admissible as an ancient document.

: Answer choice B is correct. The brother's statement is hearsay, which is an out-of-court statement introduced to prove the truth of the matter asserted. No hearsay exception applies to allow the testimony. Answer choice A is incorrect, because, although the content would likely be admissible if offered through testimony by the grantor's brother himself as the opinion of a lay witness, the affidavit itself is inadmissible hearsay. Answer choice C is incorrect because the document is not an official record made in the normal course of a business activity. Answer choice D is incorrect because the document does not qualify as an ancient document because it is not at least twenty years old.


In financial straits and needing $4,500 immediately, a nephew orally asked his uncle for a $4,500 loan. The uncle replied that he would lend the money to the nephew only if the nephew's mother "guaranteed" the loan. At the nephew's suggestion, the uncle then telephoned the nephew's mother, told her about the loan, and asked if she would "guarantee" it. She replied, "Surely. Lend my son the $4,500 and I'll repay it if he doesn't." The uncle then lent $4,500 to the nephew, an amount the nephew orally agreed to repay in six weeks. The next day, the nephew's mother wrote to him and concluded her letter with the words, "Son, I was happy to do you a favor by promising your uncle I would repay your six-week $4,500 loan if you don't. /s/ Mother." Neither the nephew nor his mother repaid the loan when it came due and the uncle sued the mother for breach of contract. In that action, the mother raised the Statute of Frauds as her only defense.

Will the mother's Statute of Frauds defense be successful?


A. No, because the amount of the loan was less than $5,000.B. No, because the mother's letter satisfies the Statute-of-Frauds requirement.C. Yes, because the mother's promise to the uncle was oral.D. Yes, because the nephew's promise to the uncle was oral.

Answer choice B is correct. There is a Statute of Frauds requirement in cases of promises to answer for the debt of another. But the memorandum sufficient to satisfy the statute needn't be written at the time of the making of the promise, nor need it be a writing addressed to the promisee. In this case, the mother's letter to her son, the nephew, satisfies the requirement of the Statute of Frauds. Answer choice A is incorrect because the amount of the loan is irrelevant here. The writing requirement arises under the suretyship provision of the Statute of Frauds. A person selecting this answer might be thinking of the $5,000 limit which may trigger a writing requirement in cases involving sales of goods under the UCC. The UCC does not apply here, since this is not a sale of goods. As to the enforceability of the mother's promise, the Statute of Frauds requires a writing in cases of promises to answer for the debt of another. Answer choice C is incorrect for the reasons given in correct answer choice B. Answer choice D is incorrect because the issue in this case is not the enforceability of the nephew's promise, but the enforceability of the mother's promise. And even if the issue did involve the nephew's promise, there is no Statute of Frauds requirement that a promise to repay a debt be in writing. The foregoing NCBE MBE question has been modified to reflect current NCBE stylistic approaches; the NCBE has not reviewed or endorsed this modification.

The Federal Automobile Safety Act establishes certain safety and performance standards for all automobiles manufactured in the United States. The Act creates a five-member "Automobile Commission" to investigate automobile safety, to make recommendations to Congress for new laws, to make further rules establishing safety and performance standards, and to prosecute violations of the act. The chairman is appointed by the President; two members are then selected by the President pro tempore of the Senate, and two by the Speaker of the House of Representatives. A minor United States car manufacturer seeks to enjoin enforcement of the Commission's rules.

The best argument that the manufacturer can make is that


A. Legislative power may not be delegated by Congress to an agency in the absence of clear guidelines.B. The commerce power does not extend to the manufacture of automobiles not used in interstate commerce.C. The manufacturer is denied due process of law because it is not represented on the Commission.D. The Commission lacks authority to enforce its standards because not all of its members were appointed by the President.

Answer choice D is correct. The statute is unconstitutional because it allows four of the five commissioners to be appointed by legislators. Congress may decide who appoints inferior executive officials. “Inferior” officials are those supervised by Senate-confirmed appointees. Such power may be vested in the President, the judicial branch, or the heads of executive branch departments. It may not be vested in the legislative branch, as Congress may not itself appoint members of a body with administrative or enforcement powers. Answer choice A is incorrect because it wrongly suggests that this statute involves the delegation of legislative power. This statute governs the appointment of executive branch officials. Answer choice B is incorrect because it wrongly invokes the Commerce Clause. Answer choice C is incorrect because it wrongly suggests that a regulated industry has a constitutional right to representation on the commission that regulates it.

A plaintiff sued a defendant for fraud. After verdict for the plaintiff, the defendant talked with a juror about the trial.

The defendant's motion for a new trial would be most likely granted if the juror is willing to testify that he voted for the plaintiff because he


A. misunderstood the judge's instructions concerning the standard of proof in a fraud case.B. was feeling ill and needed to get home quickly.C. relied on testimony that the judge had stricken and ordered the jury to disregard.D. learned from a court clerk that the defendant had been accused of fraud in several recent lawsuits.

Answer choice D is correct. Plain errors--those that affect substantial rights--are grounds for reversal even if the issue was not raised before the trial judge. Plain error is invoked to prevent a miscarriage of justice or to preserve the integrity and the reputation of the judicial process. Therefore, the fact that a juror based his verdict on evidence from an outside source that would have been inadmissible if offered at trial clearly threatens the integrity of the judicial process and is likely to result in a miscarriage of justice. Thus, answer choice D is the best answer. Answer choice A is incorrect because, during an inquiry into the validity of a verdict, a juror generally may not testify as to any matter or statement that occurred during the course of the jury's deliberations, such as misunderstanding an instruction. Answer choice B is incorrect because a juror may not testify as to the effect of anything upon that juror's physical or mental state that influenced that juror's judgment, such as a juror's physical illness. Answer choice C is incorrect because, during an inquiry into the validity of a verdict, a juror generally may not testify as to any matter or statement that occurred during the course of the jury's deliberations such as purposely refusing to follow a judicial order.

A state legislature received complaints from accident victims who, in the days immediately following their accidents, had received unwelcome and occasionally misleading telephone calls on behalf of medical care providers. The callers warned of the risks of not obtaining prompt medical evaluation to detect injuries resulting from accidents and offered free examinations to determine whether the victims had suffered any injuries. In response to these complaints, the legislature enacted a law prohibiting medical care providers from soliciting any accident victim by telephone within 30 days of his or her accident.

Which of the following is the most useful argument for the state to use in defending the constitutionality of the law?


A. Because the commercial speech that is the subject of this law includes some speech that is misleading, the First Amendment does not limit the power of the state to regulate that speech.B. Because the law regulates only commercial speech, the state need only demonstrate that the restriction is rationally related to achieving the state's legitimate interests in protecting the privacy of accident victims and in regulating the medical profession.C. The state has substantial interests in protecting the privacy of accident victims and in regulating the practice of medical care providers, and the law is narrowly tailored to achieve the state's objectives.D. The law is a reasonable time, place, and manner regulation.

Answer choice C is correct. Commercial speech—advertising and similarly economically-oriented expression—is entitled to an intermediate level of First Amendment protection. Restrictions on commercial speech are reviewed under a four-part test: i) The commercial speech must concern lawful activity and be neither false nor misleading; ii) The asserted government interest must be substantial; iii) The regulation must directly advance the asserted interest; and iv) The regulation must be narrowly tailored to serve that interest. In this context, narrowly tailored does not mean the least restrictive means available; rather, there must be a “reasonable fit” between the government’s ends and the means chosen to accomplish those ends. The law in question regulates only commercial speech, and is narrowly tailored to serve a substantial government interest. The U.S. Supreme Court has held that a law barring the solicitation of accident victims within a limited time period following an accident was narrowly tailored to serve the state's substantial interest in protecting the privacy of the victims. Answer choice A is incorrect although the law regulates only commercial speech, which is not protected by the First Amendment when misleading, because the facts state that the phone calls only "occasionally" were misleading. The First Amendment protects truthful commercial speech subject to the law. Answer choice B is incorrect because the First Amendment invalidates any law regulating such speech unless the law is narrowly tailored to serve a substantial government interest. The rational relationship between the restrictions imposed by the law and a legitimate state interest is not sufficient to satisfy this standard. Answer choice D is incorrect because the law at issue is not a time, place, and manner regulation because it restricts speech based on its content.

A defendant is charged as a principal with the rape of a 10-year-old child and with murder, under a felony murder statute, for her unintended death, which occurred during the commission of the rape. Pursuant to state law, the prosecutor timely notifies the defendant of the state's intent to seek the death penalty. The defendant challenges the prosecutor's decision on the grounds that the imposition of the death penalty for these crimes constitutes cruel and unusual punishment under the Eighth Amendment.

Should the court permit the prosecutor to seek the death penalty for these crimes?


A. Yes, as to both rape and felony murder.B. No, as to rape, but yes as to felony murder.C. No, as to felony murder, but yes as to rape.D. No, as to both rape and felony murder.

Answer choice B is correct. Capital punishment for the crime of rape, even when the victim is a child, constitutes cruel and unusual punishment under the Eighth Amendment. Accordingly, answer choices A and C are incorrect because they both would allow capital punishment for the crime of rape. Answer choice D is incorrect because major participation in the felony that resulted in the murder, plus a reckless indifference to human life, is sufficient to satisfy the culpability requirement to justify the death penalty--even absent the intent to kill. Here, although the defendant did not intend the rape victim's death, the defendant committed the violent felony that resulted in her death.

Congressional legislation authorizing marriages and divorces as a matter of federal law on prescribed terms and conditions could most easily be upheld if it


A. applied only to marriages and divorces in which at least one of the parties is a member of the armed forces.B. applied only to marriages performed by federal judges and to divorces granted by federal courts.C. implemented an executive agreement seeking to define basic human rights.D. applied only to marriages and divorces in the District of Columbia.

Answer choice D is correct. Under Article I, Section 8, Congress has supreme authority over Washington, D.C., and may legislate freely with regard to D.C. law. Outside of Washington, D.C. or federal territory, Congress does not, however, have power to legislate for the general welfare of the population. Answer choice A is incorrect because marriages and divorces are regulated according to state law, regardless of whether one of the parties was a member of the armed forces. Answer choice B is incorrect because the officiant of the ceremony is irrelevant. Answer choice C is incorrect because such an executive agreement would still not take marriages and divorces out of state courts.

A defendant was charged in state court with two crimes: burglary and the lesser included offense of criminal trespass. The defendant entered a guilty plea to criminal trespass. After determining that the defendant's plea was voluntary and intelligent, the court, over the prosecution's objection, accepted the defendant's plea and sentenced the defendant to imprisonment for the maximum time permitted by statute. The defendant then moved to dismiss the burglary charge on the grounds that prosecution of this offense violated the Double Jeopardy Clause. Again, over the prosecution's objection, the court granted the defendant's motion. On appeal, the prosecution has challenged the trial court's actions as unconstitutional.

Should the appellate court rule in favor of the prosecution?


A. Yes, because the court's acceptance of the defendant's plea without the prosecution's assent violates the Due Process Clause of the Fourteenth Amendment.B. Yes, because prosecution of the burglary charge did not violate the Double Jeopardy Clause of the Fifth Amendment as applied to the state through the Fourteenth Amendment.C. No, because the court determined that the defendant's plea to the criminal trespass charge was both voluntary and intelligent and because the criminal trespass charge was a lesser included offense of burglary.D. No, because the prosecution may not appeal the dismissal of charges against a defendant without violating the Double Jeopardy Clause of the Fifth Amendment as applied to the state through the Fourteenth Amendment.

Answer choice B is correct. Generally prosecution of a greater crime, such as burglary, after the defendant has been sentenced with regard to a lesser included offense, such as criminal trespass, which is based on the same conduct as the greater offense, violates the Double Jeopardy Clause. However, the Double Jeopardy Clause does not prohibit such prosecution where the defendant had been charged with the greater offense at the time that the defendant pleads guilty to the lesser included offense. For this reason, answer choice D is incorrect. Answer choice A is incorrect because, although rare, the court may accept a defendant's guilty plea without the prosecution's assent to the plea. In addition, the Due Process Clause protects a defendant against governmental action; it does not protect the prosecution against action by a court with which the prosecution disagrees. Answer choice C is incorrect because, although the court did, as required by the Due Process Clause, determine that the defendant's guilty plea was both intelligent and voluntary, the court's dismissal of the burglary charge on double jeopardy grounds was incorrect.


A plaintiff sued his insurance company for the full loss of his banquet hall by fire. The insurance company defended under a provision of the policy limiting liability to 50 percent if "flammable materials not essential to the operation of the business were stored on the premises and caused a fire." The insurance company called the keeper of the city fire inspection records to identify a report prepared and filed by the fire marshal as required by law, indicating that shortly before the fire, the fire marshal had cited the plaintiff for storing gasoline at the banquet hall.

Is the report admissible?


A. No, because it is hearsay not within any exception.B. No, because the proceeding is civil, rather than criminal.C. Yes, as a public record describing matters observed as to which there was a duty to report.D. Yes, as a record of regularly conducted activity, provided the fire marshal is unavailable.

Answer choice C is correct. The report is admissible as a public record under Rule 803(8) of the Federal Rules of Evidence. The facts of the problem indicate that the fire marshal had a legal duty to report. The fact that the fire marshal issued the citation indicates that he observed gasoline being stored in the banquet hall. As such, answer choice A is incorrect. Answer choice B is incorrect because the fact that this is a civil case does not make the report inadmissible. Public records are admissible in both civil and criminal cases. In fact, they are more broadly admissible in civil cases than in criminal cases. Although the exception for records of regularly conducted activity (Rule 803(6) of the Federal Rules of Evidence) might apply, answer choice D is incorrect because this exception does not require a showing of the unavailability of the fire marshal.

Several sites on a mountain within federal public lands are regarded as sacred to a group that for years has gathered there to perform religious ceremonies. The United States Forest Service recently issued a permit to a private developer to construct a ski facility in an area that includes the sites that are sacred to the group. The group filed suit in federal district court against the Forest Service to force cancellation of the permit. The group claimed solely that the permit violated its First Amendment right to the free exercise of religion. The Forest Service conceded that the group's religious beliefs were sincere and that the ski facility would adversely affect the group's religious practices.

In order to prevail in its First Amendment claim, what must the group show?


A. Construction of the ski facility will have a discriminatory impact on the group's religious practices in relation to the practices of other religious groups.B. The burden on the group's religious practices imposed by construction of the ski facility outweighs the government's interest in allowing the facility.C. The Forest Service can achieve its legitimate interest in allowing the ski facility by issuing a permit that is less burdensome on the group's religious practices.D. The permit issued by the Forest Service is aimed at suppressing the religious practices of the group.

Answer choice D is correct. Religious conduct is not absolutely protected, so only laws that intentionally target religious conduct are subject to strict scrutiny. The group would be most likely to prevail if it could require the state to meet the highest standard of strict scrutiny, as courts typically invalidate government action at strict scrutiny. Answer choice A is incorrect because the discriminatory impact of government action on a religious practice is insufficient to establish a violation of the free exercise clause. In order to establish a free exercise violation, the challenger must show that the government action targeted the religious practice in question. Answer choice B is incorrect because a simple benefit-burden balance of the government action at issue does not establish a violation of the Free Exercise Clause. Answer choice C is also incorrect because the government is required to select the least burdensome alternative only if a court exercises strict scrutiny to evaluate the action at issue. To require the court to meet strict scrutiny, the group must first show that the law intentionally targeted their religious conduct.


Two officers went to a man’s home to serve an arrest warrant, but no one answered the door. As they walked around the man’s house, the officers looked into his next door neighbor’s window and saw the man inside the neighbor’s kitchen drinking coffee. The officers knocked on the neighbor’s door, and when the neighbor answered, the officers informed the neighbor that they had a warrant to arrest the man. The officers pushed past the neighbor into the kitchen and arrested the man. While they were in the kitchen, the officers saw a bag of marijuana on the neighbor’s counter. The officers arrested the neighbor, and he was subsequently charged with possession of narcotics. The neighbor moved to suppress evidence of the drugs, and the prosecution argued that the evidence was admissible under the plain view exception to the warrant requirement.
Are the drugs seized in the neighbor’s kitchen likely to be admitted against the neighbor?


A. No, because the officers could not lawfully enter the neighbor’s home without his consent.B. No, because the officers did not knock and announce to the neighbor their intention to arrest the man.C. Yes, because the arrest warrant implicitly authorized the officers to take measures necessary to serve the warrant.D. Yes, because the officers entered the neighbor’s home under exigent circumstances.

Answer choice A is correct. A warrant to arrest an individual implicitly authorizes entry into the arrestee’s home to serve the warrant if the police have probable cause to believe that the arrestee is present. A police officer may not arrest a person in another person’s home without a search warrant, however, absent exigent circumstances or valid consent from the person whose home is being entered. In this case, the arrest warrant did not authorize the police to arrest the man in his neighbor’s home. Thus, the officers’ presence in the neighbor’s home was unlawful, and any evidence seized therein is inadmissible. Answer choice B is incorrect because the knock and announce rule pertains to arresting someone in that person’s own home. Even had the police knocked and announced their intention here, the entry would still be unconstitutional if the neighbor had not consented. In any event, even if the knock and announce rule been applicable, failure to knock and announce only invalidates the arrest; it does not trigger the exclusionary rule. Answer choice C is incorrect because the arrest warrant did not authorize the officers to enter the neighbor’s house, regardless of whether it was necessary to serve the warrant. Answer choice D is incorrect because the facts do not indicate that there were exigent circumstances, such as a reasonable belief of danger to the officer or destruction of the evidence.

While negligently driving his automobile, a man crashed into an automobile driven by a woman. The man said to the woman: "I will see to it that you are reimbursed for any losses you incur as a result of the accident." The man also called a physician and told him to take care of the woman, and that he, the man, would pay the bill.

If the physician discontinued treating the woman before she fully recovered and the woman brought an action against the physician for breach of contract, which of the following arguments, if any, by the physician would probably be effective in defense?


A. The woman furnished no consideration, either express or implied.B. The physician's contract was with the man and not with the woman.C. Whatever contract the physician may have had with the woman was discharged by novation on account of the agreement with the man.D. The man told the physician not to treat the woman, and the physician is a private doctor with no duty to the woman.

Answer choice D is correct. The woman has standing to enforce the agreement between the man and the physician as an intended beneficiary, unless a change in the man's intent causes her to only be an incidental beneficiary. Answer choice A is incorrect because, as an intended third-party beneficiary, the woman owes no consideration. Answer choice B is incorrect because the woman was an intended third party beneficiary of the contract. Accordingly, she would have the right to enforce the contract upon proving several factors. Answer choice C is incorrect because an intended third party beneficiary must approve any modification of the contract. The foregoing NCBE MBE question has been modified to reflect current NCBE stylistic approaches; the NCBE has not reviewed or endorsed this modification.


A husband conveyed his home to his wife for life, remainder to his daughter. There was a $20,000 mortgage on the home, requiring monthly payment covering interest to date, plus a portion of the principal.

Which of the following statements about the monthly payment is correct?


A. the wife must pay the full monthly payment.B. the wife must pay a portion of the monthly payment based on an apportionment of the value between the wife's life estate and the daughter's remainder.C. the wife must pay the portion of the monthly payment that represents interest.D. the daughter must pay the full monthly payment.

Answer choice C is correct. Life tenants have the obligation to pay taxes and interest on the mortgage (to the extent the property can produce income), and the duty not to commit waste. Answer choice A is incorrect because the wife, as life tenant, only needs to pay the portion of the monthly payment that represents interest. Answer choice B is incorrect because it presents the wrong calculation, as explained above. Answer choice D is incorrect because the daughter is not responsible for the portion of the monthly payment that represents interest.

4. (Question ID#1226)


In a civil action, the plaintiff sued a decedent's estate to recover damages for the injuries she suffered in a collision between her car and one driven by the decedent. At trial, the plaintiff introduced undisputed evidence that the decedent's car swerved across the median of the highway, where it collided with an oncoming car driven by the plaintiff. The decedent's estate introduced undisputed evidence that, prior to the car's crossing the median, the decedent suffered a fatal heart attack, which she had no reason to foresee, and that, prior to the heart attack, the decedent had been driving at a reasonable speed and in a reasonable manner. A statute makes it a traffic offense to cross the median of a highway.
In this case, for whom should the court render judgment?


A. The decedent's estate, because its evidence is undisputedB. The decedent's estate, because the plaintiff has not established a prima facie case of liability.C. The plaintiff, because the accident was of a type that does not ordinarily happen in the absence of negligence on the actor's part.D. The plaintiff, because the decedent crossed the median in violation of the statute.

nswer choice A is correct as the plaintiff's evidence that the decedent violated the statute and crossed over into her lane of traffic does establish a prima facie case of negligence. However, the decedent's estate successfully rebutted the plaintiff's evidence by providing an undisputed explanation of how the accident happened that is inconsistent with a finding of negligence (the decedent's unforeseeable heart attack made her unable to comply with the statute, or indeed with any standard of care). Answer choice B correctly states that the decedent's estate will prevail, but it misstates the legal basis for this conclusion. The plaintiff's evidence that the decedent violated the statute and crossed over into her lane of traffic does establish a prima facie case of negligence. Answer choice C is not correct because it may or may not be true that accidents of this type do not ordinarily happen in the absence of negligence, but that is beside the point. Answer choice D is incorrect for the reasons stated in correct answer choice A.

A state’s employee grievance system requires employees to submit grievances for final resolution to a panel of three arbitrators chosen by the parties from a statewide board of 13 arbitrators. In any given case, the grievant and the state may each eliminate five members, leaving a panel of three members. At the present time, the full board is composed of seven male arbitrators and six female arbitrators.
A female state employee filed a sexual harassment grievance against her male supervisor and the state. The state's attorney exercised all of her five strikes to eliminate five of the female arbitrators because she believed women, as a group, would necessarily be biased in favor of another woman who was claiming sexual harassment. The panel ruled against the employee, and she filed an action in an appropriate state court challenging the panel selection process as a gender-based denial of equal protection of the laws.
Is the employee likely to succeed in her argument that there was a constitutional violation?


A. Yes, because the gender classification used by the state's attorney in this case does not satisfy the requirements of intermediate scrutiny.B. Yes, because the gender classification used by the state's attorney in this case denies the grievant the right to a jury made up of her peers.C. No, because the gender classification used by the state's attorney in this case satisfies the requirements of the strict scrutiny test.D. No, because the gender classification used by the state's attorney in this case satisfies the requirements of the rational basis test.

Answer choice A is correct. When reviewing government action under equal protection, a court applies one of three levels of review, depending on the classification of persons concerned. In this case, the classification was based on gender, which is subject to intermediate scrutiny. Under this test, the government’s action must be substantially related to an important government interest, and there must be an “exceedingly persuasive justification” for the classification. The state’s attorney’s personal belief that women are inherently biased in favor of other women does not rise to this level. Answer choice B is incorrect because the right to a jury of one’s peers applies only to certain criminal prosecutions, and not to administrative hearings. Answer choices C and D are incorrect because gender classifications are subject to intermediate scrutiny, and not the strict scrutiny or rational basis standard. The foregoing NCBE MBE question has been modified to reflect current NCBE stylistic approaches; the NCBE has not reviewed or endorsed this modification.

A U.S. west-coast manufacturer gave a hardware retailer, who was relocating to the east coast, the following "letter of introduction" to an east-coast hardware wholesaler. "This will introduce you to my good friend and former customer who will be seeking to arrange the purchase of hardware inventory from you on credit. If you will let him have the goods, I will make good any loss up to $25,000 in the event of his default." The letter was signed by the west-coast manufacturer. The retailer presented the letter to the east-coast wholesaler, who then sold and delivered $20,000 worth of hardware to the retailer on credit. The east-coast manufacturer promptly notified the west-coast manufacturer of this sale.

Which of the following is NOT an accurate statement concerning the arrangement between the east-coast manufacturer and the west-coast manufacturer?


A. It was important to enforceability of the west-coast manufacturer's promise to the east-coast manufacturer that it be embodied in a signed writing.B. By extending the credit to the retailer, the east-coast manufacturer effectively accepted the west-coast manufacturer's offer for a unilateral contract.C. Although the west-coast manufacturer received no consideration from the retailer, the west-coast manufacturer's promise is enforceable by the east-coast manufacturer.D. The west-coast manufacturer's promise is enforceable by the east-coast manufacturer whether or not the east-coast manufacturer gave the west-coast manufacturer seasonable notice of the extension of credit to the retailer.

Answer choice D is correct. The west-coast manufacturer's letter was an offer for a unilateral contract, which could be accepted by the east-coast manufacturer's performance of granting the credit to the retailer. Notice of acceptance of a unilateral contract is required when the offeror is not likely to become aware that the act is being performed. Answer choice A is incorrect, as the promise to guaranty the debt of another must be in writing under the Statute of Frauds. Answer choice B is incorrect, as merely extending the credit is not enough. Notice of acceptance of a unilateral contract is required when the offeror is not likely to become aware that the act is being performed. Answer choice C is incorrect, as consideration by the retailer is unnecessary. What matters is whether there was consideration by the west-coast manufacturer.


An employee was up for a promotion but was passed over by his boss for a female colleague with more experience. After learning he had not received the promotion, he became angry with his boss and convinced himself that the colleague and the boss, who was married, were involved in a relationship. He therefore contacted the boss’s wife and convinced her to shoot the boss. They stated that they would not harm the colleague, as it might make their involvement too obvious. The employee provided her with a gun. The next day, the boss’s wife approached the colleague and the boss in their office parking lot. The wife, who was not an experienced shooter, shot the colleague in the arm, and then panicked and ran off. An onlooker rushed the colleague to the hospital. Although the injury was not life threatening, she contracted an infection during surgery and died the following week. A later investigation revealed that the infection was a result of medical malpractice that occurred during the surgery. The crime was eventually traced back to the employee and the boss’s wife, and they were both charged in connection with the colleague’s death.
The employee is most likely to be convicted of which of the following crimes?


A. Murder, attempted murder, and conspiracy to commit murder.B. Murder, conspiracy to commit murder, and solicitation.C. Attempted murder and conspiracy to commit murder only.D. Murder and conspiracy to commit murder only.

Answer choice A is correct. To prove a homicide, the prosecution must show that the defendant was the actual and proximate cause of the victim’s death. If the victim would not have died but for the defendant’s act, then the defendant’s act is the actual cause of the killing. To prove proximate cause, the death must be foreseeable. A defendant’s conduct is deemed to be foreseeable if death is the natural and probable result of the conduct. Actions by a third party (e.g., medical malpractice by the doctor treating the victim) are generally foreseeable. In this case, the boss’s wife was the actual and proximate cause of the colleague’s death, and thus would be guilty of murder of the colleague. Although the crimes of murder and attempted murder would usually merge, they only merge as to the same person. While the employee could be found guilty of the colleague’s murder, she could also be found guilty of the separate crime of attempting to kill the boss. The employee had the specific intent to kill the boss, and took a substantial step towards that result (approaching the boss in the parking lot and shooting the gun). The employee would be guilty as a co-conspirator. Conspiracy is an agreement between two or more persons to accomplish an unlawful purpose with the intent to accomplish that purpose. A conspirator can be convicted of both the offense of conspiracy and all substantive crimes committed by any other co-conspirators acting in furtherance of the conspiracy. In this case, the employee and the wife agreed to kill the boss, and the employee committed an overt act—providing a gun to the wife—in furtherance of the conspiracy. Thus, the employee would be guilty of both murder and conspiracy, even though the person killed was actually the colleague and not the boss. Answer choice B is incorrect because it fails to take into account the attempted murder of the boss. Answer choice C is incorrect because the employee would be guilty of murder, because the colleague’s death was a natural and probable cause of the wife’s actions. Answer choice D is incorrect because the employee would also be guilty of the attempted murder of the boss; because the boss was not actually murdered, that attempt would not merge with the actual conviction for the colleague’s murder.

A police officer had probable cause to believe that a defendant was involved in a robbery. The officer obtained a valid arrest warrant and went to the defendant's apartment to execute it. The officer decided to go to the defendant's apartment right when he believed the defendant would be home from work, so he could search the defendant's apartment before he had a chance to hide the stolen goods. When the officer arrived at the defendant's apartment, the door was ajar, but nothing else seemed out of the ordinary. The officer slowly opened the door and entered the apartment.

He walked toward the back of the apartment, and when he heard the defendant in a bedroom, pushed open the door, loudly told the defendant to freeze, and arrested him. Was the officer's arrest of the defendant valid?


A. Yes, because the officer went to the defendant's home when he had a reasonable belief that the defendant would be home.B. Yes, because the officer obtained a valid arrest warrant, which gives the officer the right to arrest the defendant in his home.C. No, because the officer failed to "knock and announce" before he entered the defendant's apartment.D. No, because the officer had no consent to enter the defendant's apartment.

Answer choice C is correct. Even with a warrant, most states and federal law mandate that a police officer who is making an arrest in a home must generally knock and announce his purpose before entering the home. Failure to do so will invalidate the arrest. Answer choice D is incorrect because a police officer does not need an individual's consent to enter his apartment, if he has a valid arrest warrant and a reasonable belief that the individual is home. Answer choice A is incorrect because, while having a reasonable belief that the individual is home is a requirement to entering an individual's home to arrest him, that is not enough, and the officer would still need to knock and announce. Answer choice B is incorrect, again, because absent exigent circumstances, an officer must knock and announce before entering an individual's apartment.


A plaintiff has sued a defendant, alleging that she was run over by a speeding car driven by the defendant. The plaintiff was unconscious after her injury and, accompanied by her husband, was brought to the hospital in an ambulance.
At trial, the plaintiff calls an emergency room physician to testify that when the physician asked the plaintiff's husband if he knew what had happened, the husband, who was upset, replied, "I saw my wife get run over two hours ago by a driver who went right through the intersection without looking."
Is the physician's testimony about the husband's statement admissible?


A. No, because it relates an opinion.B. No, because it is hearsay not within any exception.C. Yes, as a statement made for purposes of diagnosis or treatment.D. Yes, as an excited utterance.

Answer choice B is correct. The statement is offered to prove liability for the accident. As such, it is not a statement made for purposes of diagnosis or treatment. Moreover, the statement was made two hours after the accident, so it is very unlikely that the husband (who was not himself an accident victim) was under a continuous state of excitement between the time of the accident and the time he made the statement. Therefore, the statement is not admissible as an excited utterance, and no other hearsay exception applies. Answer choice A is incorrect. An out-of-court statement is not inadmissible simply because it contains an opinion. Statements of opinion by out-of-court declarants may be admitted if they qualify under a hearsay exception and otherwise satisfy the rules governing opinion testimony of in-court witnesses. This statement, however, is hearsay not within any exception and is inadmissible. Answer choice C is incorrect. The husband's statement is making an accusation of fault for the accident. Such a statement is not pertinent to the diagnosis or treatment of the plaintiff, as is required by the hearsay exception. No other hearsay exception applies, so the statement is inadmissible. As discussed with respect to answer choice B, because the husband made the statement two hours after the accident, it is very unlikely that the husband (who was not himself an accident victim) was under a continuous state of excitement between the time of the accident and the time he made the statement.

A senator makes a speech on the floor of the United States Senate in which she asserts that a federal civil servant with minor responsibilities was twice convicted of fraud by a state court. In making this assertion, the senator relied wholly on research done by her legislative assistant. In fact, due to his own carelessness, the legislative assistant failed to notice that it was in fact a different man with the same name as the civil servant who was convicted of these crimes. No legislation affecting the appointment or discipline of civil servants or the federal agency for which the federal civil servant works was under consideration at the time the senator made her speech.
The federal civil servant sues the senator and her legislative assistant for defamation. Both defendants move to dismiss the complaint.
How is the court likely to rule on the motion?


A. Grant it as to the legislative assistant, because he is protected by the freedom of speech guarantee against defamation actions by government officials based on his mere carelessness; but deny it as to the senator, because, as an officer of the United States, she is a constituent part of the government and, therefore, has no freedom of speech rights in that capacity.B. Grant it as to both defendants, because the senator is immune to suit for any speech she makes in the Senate under the Speech or Debate Clause of Article I, Section 6, and the legislative assistant may assert the senator's immunity for his assistance to her in preparing the speech.C. Deny it as to both defendants, because any immunity of the senator under the Speech or Debate Clause does not attach to a speech that is not germane to pending legislative business, and the legislative assistant is entitled to no greater immunity than the legislator he was assisting.D. Deny it as to the legislative assistant, because he is not a legislator protected by the Speech or Debate Clause; but grant it as to the senator, because she is immune from suit for her speech by virtue of that clause.

Answer choice B is correct. The Speech or Debate Clause of Article I, section 6 protects statements and conduct made by members of Congress and their aides in the regular course of the legislative process. The research provided by the legislative assistant and the Senator’s speech were both made in the regular course of the legislative process, so both parties are immune from suit. Answer choice A is incorrect because the Speech or Debate Clause does protect senators. Answer choice C is incorrect because the senator’s speech need not be related to pending legislation in order to be protected, and the legislative assistant is immune to the extent the senator would be had she done the research herself. Answer choice D is incorrect because legislative immunity does extend to congressional aides. The foregoing NCBE MBE question has been modified to reflect current NCBE stylistic approaches; the NCBE has not reviewed or endorsed this modification.

does felony murder merge with the dangerous crime?

yup

The plaintiff was crossing Main Street at a crosswalk. The defendant, who was on the sidewalk nearby, saw a speeding automobile heading in the plaintiff's direction. The defendant ran into the street and pushed the plaintiff out of the path of the car. The plaintiff fell to the ground and broke her leg. In an action for battery brought by the plaintiff against the defendant, will the plaintiff prevail?


A. Yes, because the defendant could have shouted a warning instead of pushing the plaintiff out of the way.B. Yes, if the plaintiff was not actually in danger and the defendant should have realized it.C. No, because the driver of the car was responsible for the plaintiff's injury.D. No, if the defendant's intent was to save the plaintiff, not to harm her.

Answer choice B is correct. A battery is a harmful or offensive contact with plaintiff's person caused by defendant's intentional act. The defendant intentionally pushed the plaintiff and caused her harm, thereby committing battery. However, the defendant can assert the defense of the plaintiff's implied consent; consent is implied by law in situations when action is necessary to save a person's life. If the plaintiff's life was in danger by the approaching car, then her consent to being pushed out of the way is implied. If she was not actually in danger, and the defendant should have known it (i.e., his mistake was not reasonable), then the plaintiff will prevail in the action for battery. Answer choice A is incorrect because the plaintiff's consent to the battery is implied - there would be no need for the defendant to shout first to validate his defense. Answer choice C is incorrect because the defendant was the direct cause of the plaintiff's injury. Answer choice D is incorrect because the defendant's motive for committing the battery has no bearing on his liability.

An undercover officer infiltrated a gang by posing as a gang member. While he was working undercover, the officer heard that a member of the gang had shot and killed one of the leaders of a rival gang. The next time the officer and the gang member suspected of the murder were alone, the officer brought up the shooting, indicating that he was very impressed by whoever had the courage to commit such an act. The gang member, taking the bait, bragged that he had shot the rival gang leader, but told the officer to keep the information secret. The gang member was arrested for the murder and the prosecution sought to introduce the statement made to the undercover officer.
Is the statement likely to be admitted?


A. No, because the gang member was questioned by a police officer without receiving Miranda warnings.B. No, because the officer deceived the gang member as to the officer’s true identity.C. Yes, because the gang member did not know that the undercover officer was a police officer.D. Yes, because the gang member was not subject to interrogation.

Answer choice C is correct. Miranda warnings are not required if the suspect being questioned is not aware that the interrogator is a police officer. Thus, an undercover officer may question a suspect without informing him of his rights, as was done in this case. Note, too, that Miranda warnings only apply to custodial interrogations, and in this case, the gang member was not in custody. Answer choice A is incorrect because the suspect did not know the undercover officer was a police officer, so Miranda warnings were not required. Answer choice B is incorrect because deceit or fraud by an interrogator (e.g., lying about a co-conspirator’s confession) does not itself make a confession involuntary. In this case, the officer’s deception regarding his identity did not make the confession involuntary. Answer choice D is incorrect because interrogation refers not only to express questioning, but also to any words or actions that the police know or should know are likely to elicit an incriminating response. In this case, the officer made a statement that he knew was likely to elicit an incriminating response. While the statement was arguably an interrogation, it was not a custodial interrogation. Accordingly, answer choice C is a better answer.

Two police officers stopped a car for a minor traffic violation. While one of the officers dealt with the driver and the traffic violation, the other officer talked with the passenger. Seeking to question the passenger about gang involvement, but lacking reasonable suspicions of criminal activity, the officer ordered the passenger out of the car. As the passenger exited the car, the officer saw a bulge in the passenger's coat which the officer suspected might be a gun. Upon patting down the passenger, the officer felt the handle of a revolver and removed the gun. The passenger, who was a convicted felon, was charged with the possession of a gun by a prohibited possessor. Prior to trial, the defendant sought to suppress the gun as evidence, contending that its seizure was unconstitutional.

Should the court suppress the gun as evidence?


A. Yes, because, since the car had been stopped for a traffic violation, the officer could not order the passenger, who had not committed the violation, to exit the car.B. Yes, because, since the officer lacked reasonable suspicion that the passenger was engaged in criminal activity, the officer could not pat down the passenger.C. No, because a valid traffic stop gives an officer the right to pat down a passenger.D. No, because the gun was discovered as a consequence of a valid Terry stop and frisk.

Answer choice D is correct. The traffic violation gave the police officers a valid reason for stopping the car. As part of the stop, the police officer could order the passenger to exit the vehicle. As the passenger did so, the police officer gained a reasonable suspicion that the passenger was armed and dangerous. Consequently, the officer could pat down the passenger for weapons. Answer choice A is incorrect because the police may order a passenger to exit a car that is legitimately stopped for a traffic violation, even though the passenger is not responsible for the traffic violation. Compelling the passenger to exit the car constitutes a de minimis additional intrusion. Answer choice B is incorrect because the officer's justification for ordering the passenger to exit the car arose from the traffic violation. The fact that the officer also had another reason for doing so that was not supported by a reasonable suspicion as constitutionally required does not prevent the officer from ordering the passenger to exit the car. Answer choice C is incorrect because an officer may not conduct a pat-down as a consequence of a valid traffic stop. The officer must have a reasonable suspicion that the passenger is armed and dangerous.


A plaintiff sued her employer, alleging that poor working conditions had caused her to develop a stomach ulcer. At trial, the plaintiff's medical expert testified to the cause of the plaintiff's ulcer and stated that his opinion was based in part on information in a letter the plaintiff's personal physician had written to the plaintiff's employer, explaining why the plaintiff had missed work.

When offered to prove the cause of the plaintiff's condition, is the letter from the plaintiff's doctor admissible?


A. No, because it is hearsay not within any exception.B. No, because the plaintiff's physician is not shown to be unavailable.C. Yes, because it was relied upon by the plaintiff's medical expert.D. Yes, under the business records exception to the hearsay rule.

Answer choice A is correct. The doctor's letter is not a business record under Federal Rule of Evidence 803(6), because it was not prepared in the ordinary course of regularly conducted activity. In addition, it cannot be admitted simply because an expert relies upon it. Rule 703 does allow an expert to rely on hearsay in reaching a conclusion, so long as other experts in the field would reasonably rely on such information. But the rule distinguishes between expert reliance on the hearsay and admitting the hearsay at trial for the jury to consider. Generally speaking, hearsay will not be admissible when offered only because the expert relied upon it. The probative value of the hearsay in illustrating the basis of the expert's opinion must substantially outweigh the risks of prejudice and confusion that will occur when the jury is told about the hearsay. That strict balancing test is not met in this case. There is no other exception that appears even close to being applicable (and none listed in the possible answers), so the letter is inadmissible hearsay. Answer choice B is incorrect as this fact would only be relevant if the hearsay were offered under Federal Rule of Evidence 804 as one of the "unavailability-dependent" exceptions to the hearsay rule. None of those exceptions are pertinent here. Furthermore, if the hearsay qualified as a hearsay exception under Rule 803 (e.g., if it were a business record), then the doctor's availability would be completely irrelevant, because unavailability need not be shown for those exceptions. But the letter does not qualify under any of the Rule 803 exceptions. In particular, it is not a business record because it was not prepared in the ordinary course of regularly conducted activity. Answer choice C is incorrect because the letter is hearsay if offered to prove that the plaintiff's condition was as indicated in the letter. Answer choice D is incorrect because the doctor's letter clearly does not qualify under Federal Rule of Evidence 803(6) as a business record. It is not a medical record prepared in the ordinary course of regularly conducted activity. Rather, it is a letter written by the doctor on an ad hoc occasion. Thus, the letter lacks the earmarks of regularity that are critical for admissibility under the business records exception to the hearsay rule.